You are on page 1of 158

Answer Key Section 1 Section 2: Math

Question # 1 2 3 4 5 6 7 8 9 10 11 12 13 14 15 16 17 18 19 20 Answer A B E E B D D A B A E C E D B B C D E C Estimated Difficulty Level E E E E E E M E E M M M M M M M M H H H

Section 3 Section 4: Critical Reading


Question # 1 2 3 4 5 Answer D E C B A Estimated Difficulty Level E M M M M

6 7 8 9 10 11 12 13 14 15 16 17 18 19 20 21 22 23 24

A E C E D B D D A B C B B E A C D A A

M H H E E H M M M M E E M E M H M M M

Section 5: Writing
Question # Your Answer Difficulty Level 1 B E 2 D H 3 C M 4 B M 5 E M 6 B M 7 B H 8 D H 9 C M 10 A H 11 C M 12 B M 13 B E 14 C E 15 B M Correct Answer ResultEstimated

16 17 18 19 20 21 22 23 24 25 26 27 28 29 30 31 32 33 34 35

B C E B B E C D B E A C B B D E A B C E

M M M M M M H M E H H M H E E E M H M M

Section 6: Math
Question # Your Answer Difficulty Level 1 A E 2 D E 3 D E 4 B M 5 E M 6 C M 7 C M 8 C H 9 7.5 or 15/2 E 10 9 M 11 22 E 12 3 or 6 M Or 9 or 12 13 96 M Correct Answer ResultEstimated

14 15 16 17 18

8 M 1024 M 109 H 10 H 49/5 or 9.8

Section 7: Critical Reading


Question # Your Answer Difficulty Level 1 B E 2 D E 3 B M 4 C M 5 A H 6 B M 7 A M 8 E M 9 C M 10 E M 11 D H 12 E M 13 C M 14 C H 15 A E 16 A M 17 B M 18 E M 19 C M 20 B M 21 C M 22 A E 23 E M 24 D M Correct Answer ResultEstimated

Section 8: Math
Question # Answer 1 D Estimated Difficulty Level E

2 3 4 5 6 7 8 9 10 11 12 13 14 15 16

D A C B B D D C B E B C C A E

E E E M M M M M M M M E M H H

Section 9: Critical Reading


Question # 1 2 3 4 5 6 7 8 9 10 11 12 13 14 15 16 17 18 19 Answer D B A C B C A B B E A D E B D C E E A Estimated Difficulty Level E M M M M H M H M M M M M M H H M M M

Section 10: Writing


Question # 1 2 3 4 5 6 7 8 9 10 11 12 13 14 Answer D B B C E D C C B A D E E E Estimated Difficulty Level E M M M M M E M M H H M H H

Explanation Section 1: Essay


Essay Prompt: Think carefully about the issue presented in the following excerpt and the assignment below. People who like to think of themselves as tough-minded and realistic tend to take it for granted that human nature is selfish and that life is a struggle in which only the fittest may survive. According to this view, the basic law by which people must live is the law of the jungle. The fittest are those people who can bring to the struggle superior force, superior cunning, and superior ruthlessness. Adapted from S.I. Hayakawa, Language in Thought and Action Do people have to be highly competitive in order to succeed? Plan and write an essay in which you develop your point of view on this issue. Support

your position with reasoning and examples taken from your reading, studies, experience, or observations. Sample Essay - Score of 6 Often, people are judged by their accomplishments. Different groups look at what an individual has done, & judge that person from only what they see whether this person is fit for this job, or that specialty track. This is the everbasic competition. It is all around us. Jobs, schools, & wars depend on it. The strong survive for one reason. They are able to adapt & overcome. They will run headfirst into a problem, then back up and look at it from a different angle. The weak will run into the problem the same way, but after backing up, will run into the problem again & again until they get too frustrated to continue or ask a stronger individual for help. The stronger, however, will be long past that problem and onto a more challenging one. This point was displayed during World War II & the development of the jet engine. The US & Great Britan were having great problems getting all the bugs worked out from uncontrollable thrust amounts & out of control speeds, to complete & total engine detonation. However, the German Luftwaffe had already perfected the engine & had put it to work in the Me 262. The allies were being destroyed by an enemy that they had nothing to match with. Allied planes were being shot down in massive numbers, & there was nothing anybody could do, because the US jet engine was far from finished. In response, new tactics were developed. Instead of just hoping to get a lucky shot & running into that wall, the Allies found a way around the wall. The idea was that during takeoff & landing, the 262 was extremely vulnerable, because it had to slow down dramatically to be able to land. Because of this, the Allied planes could now not only catch, but destroy many 262s that would have otherwise been impossible to destroy. This new tactic won the air war over the skies of Europe. Ever since man has been around, they have been making tools. From the most primitive spear, little more than a sharp stick, to the most technologically advanced military fighter, the F-22 Raptor, man has been making devices to help them for millions of years. It is the strong who in the first place develop these tools. The weak will be quickly killed off in attacks, while the strong & agile will quickly improvise something, then revise it later to make it better.

Score Explanation This outstanding essay effectively and insightfully develops a point of view that "The strong survive for one reason. They are able to adapt & overcome." The writer demonstrates outstanding critical thinking by focusing on the clearly appropriate and skillfully detailed example of how Allied forces during World War II undermined German air supremacy. The essay smoothly progresses through a chronological account of how the U.S. and Great Britain, "being destroyed by an enemy that they had nothing to match with" due to their own delay in perfecting a jet engine, developed ingenious "new tactics" for taking advantage of the way in which the German Me 262 "was extremely vulnerable." The essay consistently uses a varied, accurate, and apt vocabulary ("The US & Great Britan were having great problems getting all the bugs worked out, from uncontrollable thrust amounts & out of control speeds, to complete & total engine detonation") and demonstrates meaningful variety in sentence structure ("The idea was that during takeoff & landing, the 262 was extremely vulnerable, because it had to slow down dramatically to be able to land. Because of this, the Allied planes could now not only catch, but destroy many 262s that would have otherwise been impossible to destroy. This new tactic won the air war over the skies of Europe"). Demonstrating clear and consistent mastery, this essay receives the highest score of 6. Sample Essay - Score of 5 Where there is the will, there is a way. The people of todays society know that in order to succeed, they need to fight for what they want. Whether it be in schools, sport teams, or the work place, competition is key for survival. The common school in the United States is stereotypic of a place where students undergo stress, hardship, and pressure. The acedemic world is no pleasant place for children. Day after day young eager students fight for the grade and the path of their future. There is no wonder why it is so common to find teenagers at home behind their desks studying into the late hours of the night. They are all brainwashed. Brainwashed by their parents and their teachers. Teachers tell them that in order to get into a good college, they need to devote themselves to the book. Sleep doesnt matter, friends dont matter, they need to study. In too many cases, we find that almost brilliant teenagers are rejected from well respected schools due to lack of extra-

curricular activity. The academic field brings evident competition that causes the evident stress of the every day student. Hard working, stressed out kids often time turn to sports as a stress releaver. However, in many cases, it is unclear to them the amount of stress and grief will be added to what they now have. Something about succeeding in sports is more gratifying than anything else. In many cases, people will devote themselves to being the star athlete, or the strongest kid on the team. This competition, however, brings nothing more than overwhelming distress and anxiety. To get a job is one of the most rewarding things in the world. Too often than not people in search of the perfect career envy those who they seem to think already have it. Frequently, people with those types of jobs spend their life devoting every waking hour to getting that raise, or making the boss happy. Midlife crisess are clear examples of how stress from competition builds up throughout the working environment. The modern society we live in is one that exemplifies survival of the fitest. In order to succeed, you must fight. Score Explanation This essay demonstrates reasonably consistent mastery by effectively developing the point of view that "Whether it be in schools, sport teams, or the work place, competition is key for survival." Demonstrating strong critical thinking, the writer uses appropriate reasoning and examples to describe the role competition plays in each of these three areas. The essay is well organized, with strong emphasis on competition's negative effects and coherent progression from one area to the next ("The academic field brings evident competition that causes the evident stress of the every day student. Hard working, stressed out kids often time turn to sports as a stress releaver"). This essay uses appropriate vocabulary and demonstrates variety in sentence length and structure. ("Something about succeeding in sports is more gratifying than anything else. In many cases, people will devote themselves to being the 'star athlete,' or the 'strongest kid on the team.' This competition, however, brings nothing more than overwhelming distress and anxiety.") However, some lapses in specificity of support appear as the essay moves toward its conclusion. Therefore, to merit a higher score, the essay needs to provide further focused and specific evidence to more insightfully

and completely develop the point of view. This effective essay receives a score of 5. Sample Essay - Score of 5 "Survival of the fittest is a principle used today in our fast-pace society. It is human nature to have a desire to succeed; and, along with that comes the need for competition. Competition is a neccesity illustrated in all aspects of life from international affairs to personal struggles in daily life. Competition is forced to exist in order to succeed. This dates back to the late 1400s when Christopher Columbus desired an easier and quicker route to the West Indies. Around that time, certain countries had recently discovered new lands with riches and spices; however, it took a long time to transport these goods. Through competitiveness, Columbus desired to find a quick access to these newly found riches. In doing so, he discovered the American continents and completely changed history. As in history, competitiveness is illustrated in literature as well. In Arthur Millers, The Crucible, Abigail Williams has a desire to gain the love of John Proctor. She will go to any measure to succeed. Her competitiveness causes the destruction of a town and the killing of many innocent people. In this case, competitiveness is used in a destructive form. However neccesary competition is, there is also a negative side as well. Competition is prevailant in education from the time a student begins elementary school. Students are competing to draw the best picture, run the fastest at recess, do the best on a spelling test, and so on. This competitiveness remains with a student even in college. In order to be accepted into college you must be competitive. A certain grade point average and service are required. Education has excelled due to competition. Technology has provided many opportunities to better education. As much as competition is needed, there is a limit. When that limit is reached, problems are developed. Competition has provided society with numerous opportunities and has aided people in succeeding. Survival of the fittest prevails yet there must be exceptions to truly succeed. Score Explanation

This effective essay demonstrates strong critical thinking in developing the point of view that "Competition is a neccesity illustrated in all aspects of life from international affairs to personal struggles in daily life." Constructing a coherent progression of ideas, the writer repeatedly demonstrates the pervasiveness of competition using examples from history, literature, and education. A series of well organized paragraphs presenting positive and negative examples of competition supports the writer's concluding idea that "'Survival of the fittest' prevails yet there must be exceptions to truly succeed." Using appropriate vocabulary, this essay exhibits facility in the use of language ("Around that time, certain countries had recently discovered new lands with riches and spices; however, it took a long time to transport these goods"). Although the evidence provided is appropriate to effectively develop the position, to merit a score of 6, this essay needs to more insightfully develop the point of view by providing further focused and detailed support. Overall, the essay shows reasonably consistent mastery and receives a score of 5. Sample Essay - Score of 4 Many people feel that in order to survive in this world, they must be ruthless and do anything they can to get what they want. To some extent, this is true. While growing up, kids are taught that they have to work hard in order to succeed. This is true for anyone but that doesnt mean people have to be ruthless in order to get it. However, if a new idea is brought up in society today, such as a new invention, the person has to be persistent in making sure this new invention is recognized. Otherwise, it could go unnoticed and a great opportunity could be missed. Many business people who want the high class jobs in New York search all over the city, going through interview after interview to find a job with high pay. This person, of course, has a lot of competition ahead of him. This type of competition happens everywhere, everyday, and in almost every field of work there is. Competition happens everywhere, everyday, and in almost every field of work there is. Competition in sports, for example, has increased tremendously over the years. In sports such as football, people want the biggest, fastest, most athletic people they can find. This is because coaches want to have the best team and the players want to be on the best team. Coaches want the best and only the best so it could definately be said in this case that only the strongest survive. It is obvious that if you have plenty of money, you will go far in life because you can basically buy or do anything you please, to some extent. Once a player gets signed to a team such as the NFL, they begin to get payed

for playing the game. They receive millions of dollars in only a matter of a couple of years. The better the player they are, they more publicity they get, which will eventually lead to more money because people will pay to see them and even dress like them. In this case, only the best players survive. If you become slack and make the team have a bad season, then you are dropped and the coaches go on to find bigger and better players. This does prove the point that only the strongest survive. If you are not aggressive in getting what you want, then you may never get there. Life is a game. Some win and some lose. It all depends on the way people perceive life. Some people need lots of money in order to think that they have succeeded in life. Others just need a loving family. Score Explanation This competent essay develops the writer's point of view that "only the strongest survive. If you are not aggressive in getting what you want, then you may never get there." The writer demonstrates competent critical thinking by supporting the position that "competition happens everywhere, everyday, and in almost every field of work there is." The essay demonstrates some progression of ideas by first using less complete examples of "a new invention" and "high class jobs in New York," then moving to a central example of sports competition that is adequately elaborated though still somewhat general ("only the best players survive. If you become slack and make the team have a bad season, then you are dropped and the coaches go on to find bigger and better players"). The essay exhibits adequate facility in the use of language by using generally appropriate vocabulary ("Competition in sports, for example, has increased tremendously over the years. In sports such as football, people want the biggest, fastest, most athletic people they can find") and displays some variety in sentence structure ("If you are not aggressive in getting what you want, then you may never get there. Life is a game. Some win and some lose"). To merit a higher score, this essay needs to demonstrate clearer focus by providing further detailed evidence to more evenly and effectively develop the point of view. The response demonstrates adequate mastery and receives a score of 4. Sample Essay - Score of 4 I disagree with Hayakawas statement, it is not the strongest and fittest that prove to be most sucessful, nor is it the most skilled, but instead those with

the right mindset. Many forms of martial arts require the student to believe in themselves to perform tasks they thought they were unable to do. Political votes do not generally reflect the nominees ability, so much as personality. If a great presidential candidate was not approachable and an ok one was a real nice guy. The nice guy would win every time. If you look at sports there has been a great many talented teams thwarted by less cocky, more focused and less talented teams. People that have guided history to where it stands today, were all normal people, no superhuman strengths, not many above average scholars, Einstein made Cs in high school, along with half the past presidents and many other historical figures Im sure. Sure sometimes strength comes in handy, but in the majority of cases, it is the plan, or group bonding or moreover the group or one persons mindset that will determine how well he or she does. A friend of mine is a swim coach, and she said to me not to long ago, ninety percent of swimming, is in your head. Her swim team is very successful, and they live by this, the past few years theyve ranked highly on a national level. They know that every day in practice theyre only working on ten percent of their game, and that to earn the other ninety they must believe in themselves. So, the fittest, strongest, and smartest may survive, but theyre not the only ones who will, and they will not always come out on top. Score Explanation This generally focused response demonstrates adequate mastery by establishing a point of view that success depends less on the qualities described in the quotation than on "the right mindset," and that "the fittest, strongest, and smartest may survive, but they're not the only ones who will, and they will not always come out on top." The writer displays competent critical thinking by selecting adequate examples and organizing them to establish a progression of ideas that supports this position. The writer touches on examples of martial arts, politics, sports, and history, concluding with a more extended personal example of a swimming team's success that illustrates the assertion that "'ninety percent of swimming, is in your head.'"

The essay remains focused, creating a sense of overall coherence despite the lack of development of most of the examples provided. While the essay exhibits adequate facility in the use of language by using generally appropriate vocabulary ("Political votes do not generally reflect the nominees ability, so much as personality"), it is inconsistent ("Sure sometimes strength comes in handy, but in the majority of cases, it is the plan, or group bonding or moreover the group or one person's mindset that will determine how well he or she does"). Therefore, to achieve a higher score, this essay needs to demonstrate more consistent facility in the use of language, as well as provide additional focused and detailed evidence to more effectively and evenly develops the writer's point of view. The response receives a score of 4. Sample Essay - Score of 3 In todays society, people have to be tough. People must work for what they want, and fight for what is theirs. Todays world is dog eat dog, and its every person for themself. Years ago, the society was different. Back then people would try their best to help others and put themselves last. Today, if you dont look out for yourself, nobody else will. In the world of business everybody wants to be at the top of the ladder. The mind frame of todays working-class people is that it is okay to step on anything or anyone to make your way up that ladder of success. If everybody in one corporation is aiming for the Presidents position and only one person can get it, then the competition is going to be ruthless. Its been proven that its human nature to be selfish, therefore its likely that several of those people competing for that one position are going to get hurt. Its not fair but thats the way it is. In order to avoid getting hurt in our world today, it is important that you put your self first, and think of others later. That may sound selfish, but thats how it is. Score Explanation This response demonstrates developing mastery by establishing the point of view that competition is necessary because "Today, if you don't look out for yourself, nobody else will." The writer demonstrates some critical thinking

by selecting an appropriate example from "the world of business"; however, this evidence is inadequate to fully develop the position or establish clear coherence and progression of ideas due to reliance on generalizations ("Back then people would try their best to help others and put themselves lastThe mind frame of today's working-class people is that it is okay to step on anything or anyoneIt's been proven that it's human nature to be selfish"). Although the response exhibits developing facility in the use of language ("If everybody in one corporation is aiming for the President's position and only one person can get it, then the competition is going to be ruthless"), vocabulary is sometimes weak ("Years ago, the society was different. Back then people would try their best to help others and put themselves lastIn order to avoid getting hurt in our world today, it is important that you put yourself first, and think of others later"). Further, sentence structures are repetitive ("It's not fair but that's the way it isThat may sound selfish, but that's how it is"). To achieve a higher score, this essay needs to demonstrate stronger critical thinking by providing further focused and detailed evidence that more effectively and insightfully develop the point of view. Overall, this response remains in the inadequate category and receives a score of 3. Sample Essay - Score of 2 I think that people who have to be competitive arent to good. Yes, they strive to beat others and to be good, but they have to win. Some people who arent competive turnout to be very sucessful. One person who wasnt really competive and was very successful was Bill Gates. When he started, he was one of the only people to build computers. And now look at what happened. He is the richest person alive. He was able to get to the top because he worked hard and didnt worry to much about the competition. He turned out to be a person that everyone knows about. That would be a very successful person. In my opinion, people who have to be competitive to have success is nothing. Its a crutch that helps them get motivated. People who arent competitive can be successful. Take Bill Gates for example. He is very successful, and he wasnt that competitive. Being competitive can help you get motivated, but it can also cause you to think you can only be successful if you are competitive.

Score Explanation While this response attempts to develop the point of view that "Some people who aren't competitve turnout to be very sucessful," the lack of clear evidence to support this position results in a seriously limited essay. The writer does demonstrate some critical thinking by offering the appropriate example of Bill Gates to support this position. However, this sparse and repetitive example is insufficient to fully develop the point of view or to establish a progression of ideas ("One person who wasn't really competive and was very successful was Bill GatesHe was able to get to the top because he worked hard and didn't worry to much about the competition People who aren't competitive can be successful. Take Bill Gates for example. He is very successful, and he wasn't that competitive"). The essay displays very little facility in the use of language by consistently relying on a limited vocabulary and repetitive word choices ("He turned out to be a person that everyone knows about. That would be a very successful personHe is very successful, and he wasn't that competitive"). To achieve a higher score, this essay needs to demonstrate stronger critical thinking by maintaining focus on the position and providing further detailed, appropriate evidence that more effectively develops the point of view. This response shows little mastery and thus receives a score of 2. Sample Essay - Score of 1 In life the people that succed in life are competitive. They are the people that know what they want in life. Most people that is competitive would do anything they could to succeed and be succsful in life. The view of being highly competitive in which most people who dont think about there life and goals. Score Explanation This brief response is fundamentally lacking. While the essay presents the point of view that "In life the people that succed in life are competitive", it provides too little support to clearly develop this position. Evidence is offered as a series of vague ideas, resulting in a disjointed and practically incoherent essay. Vocabulary is limited and repetitive ("In life the people that succed in life are competitiveMost people that is competitive would do anything they could to succeed and be succsful in life"). Additionally, there are flaws in sentence structure ("The view of being highly competitive

in which most people who don't think about there life and goals") as well as errors in grammar and usage ("Most people that is competitivemost people who don't think about there life and goals"). Therefore, in order to achieve a higher score, this essay needs to demonstrate more facility in the use of language; exhibit stronger control over grammar, usage, mechanics, and sentence formation; and demonstrate stronger critical thinking by providing further focused evidence to more effectively develop the point of view. Overall, this essay demonstrates very little mastery and earns a score of 1.

Section 2: Math Section 3: No test material Section 4: Critical Reading


1 Edmund White is a ------- author: he has written novels, essays, short stories, a travel book, and a biography. (A) demonstrative (B) nebulous (C) meticulous (D) versatile (E) metaphoric ANSWERS AND EXPLANATIONS

Explanation for Correct Answer D : Choice (D) is correct. Someone who is "versatile" embraces a variety of subjects, fields, or skills. If one were to insert this term into the text, the sentence would read "Edmund White is a versatile author: he has written novels, essays, short stories, a travel book, and a biography." The colon indicates that the information listed in the second part of the sentence will support the claim made in the first part. A writer who dabbles in so many genres can certainly be described as "versatile."

Explanation for Incorrect Answer A : Choice (A) is incorrect. Someone who is "demonstrative" is given to open expressions of emotion. If one were to insert this term into the text, the sentence would read "Edmund White is a demonstrative author: he has written novels, essays, short stories, a travel book, and a biography." This term does not fit within the context of the sentence, because an emotional author will not necessarily write many different kinds of works. Explanation for Incorrect Answer B : Choice (B) is incorrect. "Nebulous" means vague or indistinct. If one were to insert this term into the text, the sentence would read "Edmund White is a nebulous author: he has written novels, essays, short stories, a travel book, and a biography." A list of the different genres in which an author writes offers no support for a claim that the author is vague. Explanation for Incorrect Answer C : Choice (C) is incorrect. "Meticulous" means extremely careful or precise. If one were to insert this term into the text, the sentence would read "Edmund White is meticulous author: he has written novels, essays, short stories, a travel book, and a biography." While a meticulous author may write a number of different things, there is no inherent connection between these two ideas. Explanation for Incorrect Answer E : Choice (D) is incorrect. "Metaphoric" means relating to metaphors or figures of speech that make an implicit comparison. If one were to insert this term into the text, the sentence would read "Edmund White is metaphoric author: he has written novels, essays, short stories, a travel book, and a biography." The fact that White has written several different types of literature is unrelated to whether or not he writes metaphorically. All of the genres listed could be written perflectly well with or without the use of metaphors. 2 The archaeologist believed the coin she unearthed was ------- evidence, unquestionable proof that the site dated to the fourth century.

(A) immaterial (B) potential (C) incriminating (D) nominal (E) indisputable ANSWERS AND EXPLANATIONS

Explanation for Correct Answer E : Choice (E) is correct. "Indisputable" means unquestionable. If one were to insert this term into the text, the sentence would read "The archaeologist believed the coin she unearthed was indisputable evidence, unquestionable proof that the site dated to the fourth century." The comma in the sentence separates two synonymous phrases. Only "indisputable evidence" has the same meaning as "unquestionable proof." Explanation for Incorrect Answer A : Choice (A) is incorrect. "Immaterial" means unimportant. If one were to insert this term into the text, the sentence would read "The archaeologist believed the coin she unearthed was immaterial evidence, unquestionable proof that the site dated to the fourth century." The sentence implies that the archaeologist's discovery is significant, so it would be illogical to call the evidence "immaterial." Explanation for Incorrect Answer B : Choice (B) is incorrect. "Potential" means possible. If one were to insert this term into the text, the sentence would read "The archaeologist believed the coin she unearthed was potential evidence, unquestionable proof that the site dated to the fourth century." A discovery that could potentially provide evidence about an archaeological site is a far cry from "unquestionable proof." Explanation for Incorrect Answer C :

Choice (C) is incorrect. Something that is "incriminating" shows evidence of a crime or fault. If one were to insert this term into the text, the sentence would read "The archaeologist believed the coin she unearthed was incriminating evidence, unquestionable proof that the site dated to the fourth century." The term "incriminating" is inappropriate in this sentence because the coin does not prove anyone guilty of wrongdoing. Explanation for Incorrect Answer D : Choice (D) is incorrect. "Nominal" means insignificant. If one were to insert this term into the text, the sentence would read "The archaeologist believed the coin she unearthed was nominal evidence, unquestionable proof that the site dated to the fourth century." "Nominal," or skimpy, evidence would never be described as "unquestionable proof." 3 Although the rigors of ballet dancing are primarily -------, this art is also emotionally and spiritually -------. (A) illusory . . taxing (B) exaggerated . . balanced (C) physical . . demanding (D) appealing . . indulgent (E) strenuous . . dubious ANSWERS AND EXPLANATIONS

Explanation for Correct Answer C : Choice (C) is correct. "Physical" means something involving or characterized by vigorous bodily activity. "Demanding" in this context means requiring much effort or attention. If one were to insert these terms into the text, the sentence would read "Although the rigors of ballet dancing are primarily physical, this art is also emotionally and spiritually demanding." The words "although" and "also" indicate that the sentence will compare two related ideas. In this case, they set up a logical contrast between ballet's physical and non-physical demands.

Explanation for Incorrect Answer A : Choice (A) is incorrect. "Illusory" refers to something that is based on illusion or that is deceptive. "Taxing" means burdensome or difficult. If one were to insert these terms into the text, the sentence would read "Although the rigors of ballet dancing are primarily illusory, this art is also emotionally and spiritually taxing." The first clause of this sentence implies that ballet is not as physically challenging as it appears, so it does not make sense to then claim that its emotional and spiritual elements are "also . . . taxing," or difficult. Explanation for Incorrect Answer B : Choice (B) is incorrect. "Exaggerated" means overstated. "Balanced" means equal or equivalent. If one were to insert these terms into the text, the sentence would read "Although the rigors of ballet dancing are primarily exaggerated, this art is also emotionally and spiritually balanced." The claim that ballet is emotionally and spiritually balanced is in no way related to the claim that its rigors are exaggerated. Explanation for Incorrect Answer D : Choice (D) is incorrect. "Appealing" means attractive or inviting, and "indulgent" means lenient. If one were to insert these terms into the text, the sentence would read "Although the rigors of ballet dancing are primarily appealing, this art is also emotionally and spiritually indulgent." It makes little sense to describe the rigors of ballet as attractive, especially in connection to a claim about ballet's emotional and spiritual "indulgence." Explanation for Incorrect Answer E : Choice (E) is incorrect. "Strenuous" means requiring great effort, energy, and exertion. "Dubious" means fraught with uncertainty or doubt. If one were to insert these terms into the text, the sentence would read "Although the rigors of ballet dancing are primarily strenuous, this art is also emotionally and spiritually dubious." It is somewhat redundant to describe rigors as strenuous, and this claim is unrelated to the assertion that ballet is emotionally and spiritually "dubious," or doubtful.

4 Studies of ------- among turtles are sometimes ------- by the fact that the subjects live so long that researchers retire before the studies can be completed. (A) extinction . . enhanced (B) longevity . . hampered (C) behavior . . belied (D) mortality . . bolstered (E) reproduction . . confirmed ANSWERS AND EXPLANATIONS

Explanation for Correct Answer B : Choice (B) is correct. "Longevity" means of great age or great span of life. "Hampered" means to impede in motion or progress. If one were to insert these terms into the text, the sentence would read "Studies of longevity among turtles are sometimes hampered by the fact that the subjects live so long that researchers retire before the studies can be completed." The sentence explains that the lengthy lifespan of turtles can cause problems for researchers. "Longevity" fits the description "lives so long." That turtles outlive the careers of the scientists studying them would certainly hamper, or impede, the study. Explanation for Incorrect Answer A : Choice (A) is incorrect. "Extinction" means the process of becoming extinct. "Enhanced" means improved, or advanced in qualities. If one were to insert these terms into the text, the sentence would read "Studies of extinction among turtles are sometimes enhanced by the fact that the subjects live so long that researchers retire before the studies can be completed." It is illogical to claim that a study could be enhanced, or improved, by the inability of researchers to complete it. Explanation for Incorrect Answer C : Choice (C) is incorrect. "Behavior" means the manner in which something functions. "Belied" means pictured falsely or misrepresented. If one were to

insert these terms into the text, the sentence would read "Studies of behavior among turtles are sometimes belied by the fact that the subjects live so long that researchers retire before the studies can be completed." It makes some sense to say that a study could be belied, or misrepresented, by the problem of researchers retiring, but it is unclear why a behavior study would be affected by this problem. Explanation for Incorrect Answer D : Choice (D) is incorrect. "Mortality" means death rate. "Bolstered" means supported or propped up with. If one were to insert these terms into the text, the sentence would read "Studies of mortality among turtles are sometimes bolstered by the fact that the subjects live so long that researchers retire before the studies can be completed." A study on turtle mortality would not be bolstered, or propped up by, the retirement of the researcher. Explanation for Incorrect Answer E : Choice (E) is incorrect. "Reproduction" means the act of reproducing. "Confirmed" means strengthened or made valid. If one were to insert these terms into the text, the sentence would read "Studies of reproduction among turtles are sometimes confirmed by the fact that the subjects live so long that researchers retire before the studies can be completed." The fact of researchers retiring would do nothing to confirm, or prove, anything about reproduction. 5 A model of ------- behavior, Cunningham never ate or drank to excess. (A) temperate (B) laconic (C) duplicitous (D) aesthetic (E) voluble ANSWERS AND EXPLANATIONS

Explanation for Correct Answer A :

Choice (A) is correct. "Temperate" in this context refers to moderation in the indulgence of appetite. If one were to insert this term into the text, the sentence would read "A model of temperate behavior, Cunningham never ate or drank to excess." The comma indicates that the second half of the sentence will illustrate Cunningham's behavior. Someone who never consumes too much does indeed exhibit "temperate behavior." Explanation for Incorrect Answer B : Choice (B) is incorrect. Someone who is "laconic" speaks using a minimum of words. If one were to insert this term into the text, the sentence would read "A model of laconic behavior, Cunningham never ate or drank to excess." A tendency for reserved speech has nothing to do with a moderate appetite. Explanation for Incorrect Answer C : Choice (C) is incorrect. "Duplicitous" behavior is marked by deceptiveness. If one were to insert this term into the text, the sentence would read "A model of duplicitous behavior, Cunningham never ate or drank to excess." Deceptiveness is not necessarily related to eating and drinking habits. Explanation for Incorrect Answer D : Choice (D) is incorrect. "Aesthetic" means appreciative of beauty. If one were to insert this term into the text, the sentence would read "A model of aesthetic behavior, Cunningham never ate or drank to excess." One's appreciation of beauty does not necessarily affect one's appetite. Explanation for Incorrect Answer E : Choice (E) is incorrect. Someone who is "voluble" is marked by a ready flow of speech. If one were to insert this term into the text, the sentence would read "A model of voluble behavior, Cunningham never ate or drank to excess." It is illogical to claim that a talkative person always eats and drinks in moderation. 6

The entrepreneur had a well-deserved reputation for -------, having accurately anticipated many changes unforeseen by established business leaders. (A) prescience (B) sincerity (C) avarice (D) complicity (E) mendacity ANSWERS AND EXPLANATIONS

Explanation for Correct Answer A : Choice (A) is correct. "Prescience" means foresight. If one were to insert this term into the text, the sentence would read "The entrepeneur had a welldeserved reputation for prescience, having accurately anticipated many changes unforeseen by established business leaders." The part of the sentence following the comma describes the missing term. It is logical for a person who can frequently predict "unforeseen" events to develop a reputation for "prescience." Explanation for Incorrect Answer B : Choice (B) is incorrect. "Sincerity" means genuineness and honesty. If one were to insert this term into the text, the sentence would read "The entrepeneur had a well-deserved reputation for sincerity, having accurately anticipated many changes unforeseen by established business leaders." Sincerity is not linked to one's ability to anticipate changes. Explanation for Incorrect Answer C : Choice (C) is incorrect. "Avarice" means greed. If one were to insert this term into the text, the sentence would read "The entrepeneur had a welldeserved reputation for avarice, having accurately anticipated many changes unforeseen by established business leaders." The entrepreneur may be a greedy person, but this sentence describes a different trait. Explanation for Incorrect Answer D :

Choice (D) is incorrect. "Complicity" refers to a person's participation in a questionable act or a crime. If one were to insert this term into the text, the sentence would read "The entrepeneur had a well-deserved reputation for complicity, having accurately anticipated many changes unforeseen by established business leaders." This sentence offers no evidence that the entrepreneur has a reputation for conniving behavior Explanation for Incorrect Answer E : Choice (E) is incorrect. "Mendacity" means untruthfulness. If one were to insert this term into the text, the sentence would read "The entrepeneur had a well-deserved reputation for mendacity, having accurately anticipated many changes unforeseen by established business leaders." The description in the second half of this sentence does not support the claim that the entrepreneur was dishonest. 7 Scientists require observable data, not -------, to support a hypothesis; sound science is grounded in ------- results rather than speculation. (A) induction . . diminutive (B) experimentation . . pragmatic (C) intuition . . fiscal (D) bombast . . theoretical (E) conjecture . . empirical ANSWERS AND EXPLANATIONS

Explanation for Correct Answer E : Choice (E) is correct. "Conjecture" means guesswork or predicting from incomplete or uncertain evidence. "Empirical" means by experiment or according to experience. If one were to insert these terms into the text, the sentence would read "Scientists require observable data, not conjecture, to support a hypothesis; sound science is grounded in empirical results rather than speculation." It makes sense to contrast "observable data" with "conjecture" and "empirical" with "speculation." Scientific criteria usually

require hypotheses to be grounded in "empirical results" and "observable data," not "conjecture" and "speculation." Explanation for Incorrect Answer A : Choice (A) is incorrect. "Induction" means the process of deriving general principles from particular facts or instances. "Diminutive" means tiny. If one were to insert these terms into the text, the sentence would read "Scientists require observable data, not induction, to support a hypothesis; sound science is grounded in diminutive results rather than speculation." It makes no sense to say that sound science is grounded in "diminutive," or tiny, results. Nor does it make sense to contrast tiny results with "speculation." Explanation for Incorrect Answer B : Choice (B) is incorrect. "Experimentation" means the act of testing under controlled conditions. "Pragmatic" means practical. If one were to insert these terms into the text, the sentence would read "Scientists require observable data, not experimentation, to support a hypothesis; sound science is grounded in pragmatic results rather than speculation." "Experimentation" can provide "observable data"; the two are not opposites. Explanation for Incorrect Answer C : Choice (C) is incorrect. "Intuition" means immediate cognition. "Fiscal" means relating to finances. If one were to insert these terms into the text, the sentence would read "Scientists require observable data, not intuition, to support a hypothesis; sound science is grounded in fiscal results rather than speculation." While "intuition" is clearly distinguished from "observable data," science is supposed to rely on the physical results of experimentation regardless of the "fiscal," or financial, results the experimentation may involve. Explanation for Incorrect Answer D : Choice (D) is incorrect. "Bombast" means pompous speech or writing. "Theoretical" means based on theory. If one were to insert these terms into the text, the sentence would read "Scientists require observable data, not bombast, to support a hypothesis; sound science is grounded in theoretical results rather than speculation." "Theoretical results" can be described as

speculative; however, theoretical results and speculation are not mutually exclusive. 8 The director complained that the sitcoms theme song was downright -------, having no more pep and vigor than a -------. (A) tedious . . jingle (B) inchoate . . lullaby (C) lugubrious . . dirge (D) facetious . . ballad (E) sprightly . . eulogy ANSWERS AND EXPLANATIONS

Explanation for Correct Answer C : Choice (C) is correct. "Lugubrious" means mournful, and a "dirge" is a song of grief. If one were to insert these terms into the text, the sentence would read "The director complained that the sitcom's theme song was downright lugubrious, having no more pep and vigor than a dirge." In this sentence, the information following the comma supports the claim preceding the comma. The director's complaint that the song was "lugubrious" is perfectly supported by calling it a "dirge." Explanation for Incorrect Answer A : Choice (A) is incorrect. "Tedious" means boring. "Jingle" is a piece of light singsong verse or rhyme. If one were to insert these terms into the text, the sentence would read "The director complained that the sitcom's theme song was downright tedious, having no more pep and vigor than a jingle." A "jingle" is a light or catchy song, thus it makes no sense to complain that a song has no more pep than a jingle. Explanation for Incorrect Answer B : Choice (B) is incorrect. "Inchoate" means in an initial or early stage. "Lullaby" is a soothing song with which to lull a child to sleep. If one were

to insert these terms into the text, the sentence would read "The director complained that the sitcom's theme song was downright inchoate, having no more pep and vigor than a lullaby." It does not make complete sense to complain that a song lacks "pep and vigor" because it is in its early stages of development. Explanation for Incorrect Answer D : Choice (D) is incorrect. "Facetious" means joking or humorous. "Ballad" means a song of a romantic or sentimental nature. If one were to insert these terms into the text, the sentence would read "The director complained that the sitcom's theme song was downright facetious, having no more pep and vigor than a ballad." A facetious song would most likely be witty or joking, and would not likely be lacking pep. Furthermore, many ballads could be described as vigorous. Explanation for Incorrect Answer E : Choice (E) is incorrect. "Sprightly" means full of spirit and vitality. "Eulogy" is a speech praising someone who has died. If one were to insert these terms into the text, the sentence would read "The director complained that the sitcom's theme song was downright sprightly, having no more pep and vigor than a eulogy." A sprightly, or lively, song would certainly have more pep than a eulogy, or funeral speech. In between school days, we gathered hazelnuts, fished, had long deer-hunting weekends, went to powwows, beaded on looms, and made quilts. I did not Line question the necessity or value of our school 5 education, but somehow I grew up knowing it wasnt the only education I would need. Im thankful for those experiences of my Anishinaabe heritage, because now I know by heart not only the national anthem, but the ancient song of the loon. I recognize not only the alpha10 bet and the parts of an English sentence, but the intricate language of a beavers teeth and tail. 9 The main idea of the passage is that the author

(A) preferred certain academic subjects over others (B) succeeded in learning to speak many foreign languages (C) valued knowledge of the natural world more than book learning (D) loved both family trips and tribal activities (E) learned many important things both in and out of school ANSWERS AND EXPLANATIONS

Explanation for Correct Answer E : Choice (E) is correct. As the author explains in the text, "No one questioned the necessity or value of our school education, but . . . it wasn't the only education I would need." References to "deer-hunting weekends" and "powwows" make it clear that the author received an important cultural education outside the classroom. Explanation for Incorrect Answer A : Choice (A) is incorrect. The author indicates that education can take on different forms but does not express a preference for any academic subject. Explanation for Incorrect Answer B : Choice (B) is incorrect. While the author claims to have learned the "song of the loon" and the "intricate language of a beaver's teeth and tail," the only spoken language mentioned in the passage is English. Explanation for Incorrect Answer C : Choice (C) is incorrect. The author implies that both forms of education described in the passage are important but does not indicate a preference for one over the other. Explanation for Incorrect Answer D : Choice (D) is incorrect. The passage compares what the author learned both in school and outdoors. "Family trips and tribal activities" account for only the latter.

10 The authors overall tone in this passage is best described as one of (A) jubilation (B) frustration (C) curiosity (D) appreciation (E) uncertainty ANSWERS AND EXPLANATIONS

Explanation for Correct Answer D : Choice (D) is correct. The author is clearly "thankful" (line 6) for having had the opportunity to supplement a school education with outdoor lessons. Explanation for Incorrect Answer A : Choice (A) is incorrect. While the author is clearly pleased to have received two complementary forms of education, the tone of the passage never elevates to a level of "jubilation," or rejoicing. Explanation for Incorrect Answer B : Choice (B) is incorrect. The author expresses "frustration" in neither content nor tone. Explanation for Incorrect Answer C : Choice (C) is incorrect. While the author seems to enjoy learning, his primary tone is not one of "curiosity." Explanation for Incorrect Answer E : Choice (E) is incorrect. The author writes with confidence, not "uncertainty."

Daily life is overflowing with mundane mental events. A paper clip gleams amid stacks of documents, a friends face shines like a beacon out of a crowd, the smell of Line freshly baked bread evokes childhood memoriesthoughts 5 and perceptions such as these flow by with monotonous ease. So it seems, anyway. Yet given what scientists know about how brains work, even the ability to perceive a paper clip on a messy desk represents an extraordinary 10 and mysterious achievement. 11 In the first paragraph, the author implicitly likens our experience of sensory impressions to (A) an emotional roller coaster (B) an unobstructed stream (C) a repeated image (D) a nostalgic reminiscence (E) a diverting daydream ANSWERS AND EXPLANATIONS

Explanation for Correct Answer B : Choice (B) is correct. Suggestions that "life is overflowing with mundane mental events" and that "thoughts and perceptions . . . flow by" produce an implicit comparison between people's daily experiences and an "unobstructed stream." Explanation for Incorrect Answer A : Choice (A) is incorrect. The author's description of "mundane" and "monotonous" thoughts directly contrasts with the turbulence associated with "an emotional roller coaster." Explanation for Incorrect Answer C :

Choice (C) is incorrect. The author lists several different images in the first paragraph but does not suggest that any of them are repetitive. Explanation for Incorrect Answer D : Choice (D) is incorrect. While the author asserts that particular perceptions can evoke old memories, it would be inaccurate to claim that all thoughts are linked to nostalgia. Explanation for Incorrect Answer E : Choice (E) is incorrect. The author implies that most daily perceptions are boring, not "diverting," or amusing. 12 The primary purpose of the passage is to (A) celebrate lifes mundane but gratifying pleasures (B) convey the overwhelming confusion of every-day life (C) explore the biological implications of a persons decisions (D) suggest the complexity of perceptual processes (E) present a scientific analysis of an automatic reflex ANSWERS AND EXPLANATIONS

Explanation for Correct Answer D : Choice (D) is correct. The passage mainly serves to highlight the "complexity of perceptual processes" that might otherwise be taken for granted. Explanation for Incorrect Answer A : Choice (A) is incorrect. While the author seems to marvel at the underlying complexity of everyday perceptions, the passage's tone is more analytical than celebratory. Explanation for Incorrect Answer B :

Choice (B) is incorrect. Far from "convey[ing] the overwhelming confusion of everyday life," the passage in fact shows how our perceptions render everyday life easy and "monotonous." Explanation for Incorrect Answer C : Choice (C) is incorrect. The passage does not discuss people's decisions, only their perceptions. Explanation for Incorrect Answer E : Choice (E) is incorrect. The passage does allude to the scientific study of perception, but it does not present actual "scientific analysis" of any kind.

The following passage is from a 1991 essay that discusses the debate over which authors should be taught in English classes. Now, what are we to make of this sputtering debate, in which charges of imperialism are met by equally passionate accusations of vandalism, in which each side Line hates the other, and yet each seems to have its share of 5 reason? It occurs to me that perhaps what we have here is one of those debates in which the opposing sides, unbeknownst to themselves, share a myopia that will turn out to be the most interesting and important feature of the whole discussion, a debate, for instance, like that of the 10 Founding Fathers over the nature of the franchise. Think of all the energy and passion spent debating the question of property qualifications, or direct versus legislative elections, while all along, unmentioned and unimagined, was the factto us so centralthat women and slaves 15 were never considered for any kind of vote. While everyone is busy fighting over what should be taught in the classroom, something is being overlooked. That is the state of reading, and books, and literature in our country, at this time. Why, ask yourself, is everyone so hot 20 under the collar about what to put on the required-reading shelf? It is because, while we have been arguing so fiercely

25

30

35

40

45

50

55

60

about which books make the best medicine, the patient has been slipping deeper and deeper into a coma. Let us imagine a country in which reading was a popular voluntary activity. There, parents read books for their own edification and pleasure and are seen by their children at this silent and mysterious pastime. These parents also read to their children, give them books for presents, talk to them about books, and underwrite, with their taxes, a public library system that is open all day, every day. In school, the children study certain books together but also have an active reading life of their own. Years later, it may even be hard for them to remember if they read Jane Eyre at home and Judy Blume 1 in class or the other way around. In college, young people continue to be assigned certain books, but far more important are the books they discover for themselves browsing in the library, in bookstores, on the shelves of friends, one book leading to another, back and forth in history and across languages and cultures. After graduation, they continue to read and in the fullness of time produce a new generation of readers. Oh happy land! I wish we all lived there. In that country of real readers, voluntary, active, selfdetermined readers, a debate like the current one over the canon would not be taking place. Or if it did, it would be as a kind of parlor game: What books would you take to a desert island? Everyone would know that the top-ten list was merely a tiny fraction of the books one would read in a lifetime. It would not seem racist or sexist or hopelessly hidebound to put Nathaniel Hawthorne on the list and not Toni Morrison 2. It would be more like putting oatmeal and not noodles on the breakfast menua choice partly arbitrary, partly a nod to the national past, and partly, dare one say it, a kind of reverse affirmative action: School might frankly be the place where one reads the books that are a little off-putting, that have gone a little cold, that you might overlook because they do not address, in readerfriendly contemporary fashion, the issues most immediately at stake in modern life but that, with a little study, turn out to have a great deal to say. Being on the list wouldnt mean so much. It might even add to a writers cachet not to be on

65

the list, to be in one way or another too heady, too daring, too exciting to be ground up into institutional fodder for teenagers. Generations of high school kids have been turned off to George Eliot 3 by being forced to read Silas Marner at a tender age. One can imagine a whole new readership for her if grown-ups were left to approach Middlemarch and Daniel Deronda with open minds, at their leisure.

1 Jane Eyre, by Charlotte Bront, is a nineteenth-century novel. Judy Blume writes contemporary young adult novels. 2 Hawthorne was a nineteenth-century American writer. Toni Morrison is a contemporary American writer. 3 George Eliot was the pseudonym of a nineteenth-century female British novelist.

13 According to the author, too much energy today is spent debating (A) how to improve the education system (B) how to make literature seem relevant (C) who the better writers are (D) what students should read in school (E) whether or not to teach classic works ANSWERS AND EXPLANATIONS

Explanation for Correct Answer D : Choice (D) is correct. The author suggests that more time should be spent reviving students' interest in reading, not debating what they should read in school.

Explanation for Incorrect Answer A : Choice (A) is incorrect. The text suggests that the frenzy regarding English curricula is misplaced, but it does not imply that there is an excessive push for system-wide change. Explanation for Incorrect Answer B : Choice (B) is incorrect. It is more likely that the author would argue for increased efforts to make literature seem relevant, and therefore appealing, to students. Explanation for Incorrect Answer C : Choice (C) is incorrect. The author refers to different writers but does not assess the relative quality of their work. Explanation for Incorrect Answer E : Choice (E) is incorrect. The author mentions several classic novels in the passage but does not imply that the merits of these books are debated. 14 In the first two paragraphs of the passage (lines 1-23), the author suggests that both sides of the debate (A) neglect a fundamental issue (B) disregard a key piece of evidence (C) ignore opposing views (D) lack a historical perspective (E) dismiss a valuable tradition ANSWERS AND EXPLANATIONS

Explanation for Correct Answer A :

Choice (A) is correct. The author asserts that the debate swirling around academic reading lists widely ignores the crucial fact that modern youth have a deteriorating relationship with books. Explanation for Incorrect Answer B : Choice (B) is incorrect. The first two paragraphs allude to various arguments, but they do not indicate that any of the arguments lack evidence. Explanation for Incorrect Answer C : Choice (C) is incorrect. The author argues that the debate ignores a key issue. At no point, however, does the passage suggest that the debate's participants "ignore" opposing viewpoints. Explanation for Incorrect Answer D : Choice (D) is incorrect. The passage contains an example drawn from United States history, but the author does not suggest that either side of the debate lacks "historical perspective." Explanation for Incorrect Answer E : Choice (E) is incorrect. The author imagines a world in which reading is a valuable domestic tradition, passed from one generation to the next, but does not claim that both sides of the debate have dismissed any traditions. 15 The author invokes the Founding Fathers (lines 9-10) chiefly in order to (A) appeal to the readers sense of patriotism (B) introduce a historical parallel (C) examine the history of legislative debate (D) remind the reader how attitudes change over time (E) suggest that progress is compatible with tradition ANSWERS AND EXPLANATIONS

Explanation for Correct Answer B : Choice (B) is correct. Lines 910 set up a "parallel," or comparison, between the modern debate about reading lists and historical debates concerning property rights and electoral procedures. All, the author argues, fail to address crucial issues. Explanation for Incorrect Answer A : Choice (A) is incorrect. A mere mention of the "Founding Fathers" may ignite a spark of "patriotism" in some readers, but the text does not suggest that this is the author's purpose in employing the phrase. Explanation for Incorrect Answer C : Choice (C) is incorrect. The author argues that the modern debate over reading lists is reminiscent of the early American debate regarding direct versus legislative elections. The role of legislation in the passage, however, does not extend beyond this reference. Explanation for Incorrect Answer D : Choice (D) is incorrect. Although the passage implies that ideas often change over time, the author's reference to "the Founding Fathers" does not indicate that attitudes regarding the state of reading in the United States will change in the future. Explanation for Incorrect Answer E : Choice (E) is incorrect. "Progress" may be compatible with certain traditions, but the author does not make such a claim at any point in the passage. 16 In line 18, state most nearly means (A) (B) (C) government territory condition

(D) scale (E) mood ANSWERS AND EXPLANATIONS

Explanation for Correct Answer C : Choice (C) is correct. "State" in this context refers to a condition or mode of being. The author suggests throughout the passage that the "state" of Americans' relationships with books is deteriorating. Explanation for Incorrect Answer A : Choice (A) is incorrect. While the term "state" refers to a mode of government in some cases, this definition is inappropriate in the context of line 18. Explanation for Incorrect Answer B : Choice (B) is incorrect. "State" can mean a territorial unit, but this meaning is unlikely within the context of the text. Explanation for Incorrect Answer D : Choice (D) is incorrect. It would be illogical to suggest that the author would refer to the "scale," or unit of measurement, of reading. Explanation for Incorrect Answer E : Choice (E) is incorrect. "State" can mean a mental or an emotional "condition," but activities such as reading cannot be described as having either. 17 In line 23, the coma represents the (A) (B) rebellion of students against traditional texts lack of enthusiasm for reading in general

(C) scarcity of books on official reading lists (D) difficulty of understanding archaic language (E) negative effects of popular media ANSWERS AND EXPLANATIONS

Explanation for Correct Answer B : Choice (B) is correct. "Comatose" can also mean lethargic or apathetic. In creating a parallel between the "coma" in line 23 and the "state of reading" in line 18, the author is suggesting that Americans' enthusiasm for books is "slipping deeper and deeper." Explanation for Incorrect Answer A : Choice (A) is incorrect. Debate participants may be "hot under the collar," but the text makes no reference to upset or rebellious students. Explanation for Incorrect Answer C : Choice (C) is incorrect. There is little evidence in the passage to support the claim that books on academic reading lists are "scarce," or in short supply. Explanation for Incorrect Answer D : Choice (D) is incorrect. There is no mention of "archaic language" at any point in the text. Explanation for Incorrect Answer E : Choice (E) is incorrect. Certain books reflect popular culture, but the author does not draw any comparisions between "popular media" and a comatose patient. 18 In lines 2427 (Let . . . pastime), the country described is noteworthy because

(A) people have allowed new interests to develop from their reading (B) parents demonstrate their enjoyment of reading (C) children learn to read at an early age (D) children and parents share many activities (E) writing is viewed as a valuable skill ANSWERS AND EXPLANATIONS

Explanation for Correct Answer B : Choice (B) is correct. Lines 2427 describe an imagined world in which children routinely observe their parents reading for "pleasure." Explanation for Incorrect Answer A : Choice (A) is incorrect. Reading may expose people to new interests or activities, but the author does not address this possibility. Explanation for Incorrect Answer C : Choice (C) is incorrect. While children who notice their parents' enjoyment of books may be more likely to read at an early age, the author makes no such claim. Explanation for Incorrect Answer D : Choice (D) is incorrect. The passage does not describe "many activities" shared between adults and children, only reading. Explanation for Incorrect Answer E : Choice (E) is incorrect. Reading and writing skills are often linked, but lines 2427 address only the former. 19 Lines 3039 (In school . . . cultures) present a model of education where students learn to

(A) value cultural diversity over tradition (B) respect the views of both sides of the debate (C) reflect critically on the nature of American schooling (D) differentiate between classic and contemporary works (E) explore the world through wide-ranging reading ANSWERS AND EXPLANATIONS

Explanation for Correct Answer E : Choice (E) is correct. In lines 3039, the author describes a "happy land" where students are encouraged to explore books that represent different eras, languages, and cultures. Explanation for Incorrect Answer A : Choice (A) is incorrect. The author encourages students to read culturally diverse texts in line 39 but does not imply that diversity is more important than tradition. Explanation for Incorrect Answer B : Choice (B) is incorrect. The selected lines construct a world in which the canon debate does not exist. Explanation for Incorrect Answer C : Choice (C) is incorrect. The students mentioned in the quoted passage may reflect critically on literature and culture, but the passage does not describe them analyzing the nature of the American educational system. Explanation for Incorrect Answer D : Choice (D) is incorrect. Although the students mentioned in the quoted passage may be able to differentiate between classic and modern works, this ability is not discussed in the passage. 20

In lines 3334, the author cites Jane Eyre and Judy Blume primarily in order to (A) propose that a love of reading might blur a commonly perceived distinction (B) show that younger readers cannot distinguish between literature of different eras (C) argue that most modern novels have no lasting impact on readers (D) observe that classic literature has great appeal for even reluctant readers (E) indicate that certain works are interchangeable ANSWERS AND EXPLANATIONS

Explanation for Correct Answer A : Choice (A) is correct. The third paragraph describes an idealized reality "in which reading was a popular voluntary activity." The author suggests that in this world students read books so voraciously that the distinction between books read for school and books read for pleasure becomes blurred. Explanation for Incorrect Answer B : Choice (B) is incorrect. Although Jane Eyre and Judy Blume novels were composed centuries apart, the passage does not mention these two authors to suggest that young readers are incapable of "distinguishing between literature of different eras." Explanation for Incorrect Answer C : Choice (C) is incorrect. Jane Eyre's popularity has endured through time, but the author does not refer to this nineteenth-century classic in an assertion that modern novels, such as those written by Judy Blume, have only shortterm appeal. Explanation for Incorrect Answer D :

Choice (D) is incorrect. Certain literary classics such as Jane Eyre may appeal to "even reluctant readers," but lines 3334 are concerned with only "voluntary, active, self-determined" readers. Explanation for Incorrect Answer E : Choice (E) is incorrect. The author suggests that students may forget the impetus behind their reading of Jane Eyre or of Judy Blume novels but does not imply that Jane Eyre and the works of Judy Blume are interchangeable. 21 In lines 35-39 (In college . . . cultures), the education illustrated is best described as (A) elitist (B) philanthropic (C) eclectic (D) methodical (E) rudimentary ANSWERS AND EXPLANATIONS

Explanation for Correct Answer C : Choice (C) is correct. "Eclectic" means made up of elements from a variety of sources. An education supplemented by the independent discovery of books "in the library, in bookstores, on the shelves of friends" and beyond would indeed be "eclectic." Explanation for Incorrect Answer A : Choice (A) is incorrect. "Elitist" means snobbish, or privileged, but the education illustrated in these lines is available to the masses. Explanation for Incorrect Answer B : Choice (B) is incorrect. It is illogical to describe the education illustrated in the passage as "philanthropic," or charitable.

Explanation for Incorrect Answer D : Choice (D) is incorrect. "Methodical" means systematic. The education described in the passage, however, is more loosely structured. Explanation for Incorrect Answer E : Choice (E) is incorrect. The education mentioned in the quoted passage is hardly "rudimentary," or basic. 22 In lines 5460 (School . . . say), the author describes a world in which schools teach books that are (A) interesting (B) celebrated (C) uncontroversial (D) not obviously relevant (E) not likely to inspire ANSWERS AND EXPLANATIONS

Explanation for Correct Answer D : Choice (D) is correct. The reading list described in lines 5460 comprises books that do not address "the issues most immediately at stake in modern life" and that thus are not obviously relevant. Explanation for Incorrect Answer A : Choice (A) is incorrect. Some readers may find the books described in lines 5460 interesting, but the author does not imply that they are particularly engaging. In fact, the author says they are off-putting and cold. Explanation for Incorrect Answer B :

Choice (B) is incorrect. The ideal literary canon described in the final paragraph may include celebrated books, but the author does not address their popularity. Explanation for Incorrect Answer C : Choice (C) is incorrect. While the passage suggests that academic reading lists have caused a good deal of controversy, the author does not discuss whether or not the books on an ideal list would be controversial. Explanation for Incorrect Answer E : Choice (E) is incorrect. The author does not indicate how inspirational the books on an ideal reading list would be. 23 Lines 60-64 (Being . . . teenagers) suggest that excluding a book from a reading list might (A) enhance the reputation of the books author (B) encourage students to protest the decision (C) influence course curricula nationwide (D) appease conservative parents (E) disappoint the books fans ANSWERS AND EXPLANATIONS

Explanation for Correct Answer A : Choice (A) is correct. Lines 6064 suggest that exclusion from the list would contribute to an author's reputation for being "daring" and "exciting." Explanation for Incorrect Answer B : Choice (B) is incorrect. Lines 6064 give no indication that students would "protest" the exclusion of any book. Explanation for Incorrect Answer C :

Choice (C) is incorrect. Though the exclusion of a book from the author's ideal canon could affect what students read in school, lines 6064 are more concerned with the influence that such an exclusion would have on a writer's reputation. Explanation for Incorrect Answer D : Choice (D) is incorrect. While "conservative parents" may support the exclusion of "heady" or "daring" authors from academic reading lists, lines 6064 make no reference to parents at all. Explanation for Incorrect Answer E : Choice (E) is incorrect. Lines 6064 say nothing about the "fans" of any particular book. 24 The main purpose of the passage is to (A) shift the focus of a debate (B) support one side in a debate (C) suggest a practical solution (D) revive a discredited idea (E) promote certain kinds of writing ANSWERS AND EXPLANATIONS

Explanation for Correct Answer A : Choice (A) is correct. The main goal of the passage is to shift the reading debate's focus from the content of academic reading lists to ways that Americans' ailing relationships with books might be improved. Explanation for Incorrect Answer B :

Choice (B) is incorrect. Instead of promoting "one side" of an existing debate, the author suggests that both sides are ignoring "the most interesting and important feature of the whole discussion." Explanation for Incorrect Answer C : Choice (C) is incorrect. The passage provides less of a practical solution to a problem than a new perspective from which to view the problem. Explanation for Incorrect Answer D : Choice (D) is incorrect. The author makes no attempt at reviving "a discredited idea" at any point in the passage. Explanation for Incorrect Answer E : Choice (E) is incorrect. The author's focus is on reading rather than on writing.

Section 5: Writing
1 Inside famed actor Lily Langtrys private railroad car were a drawing room with a piano, bath fixtures of silver, and there were draperies trimmed with Brussels lace. (A) there were draperies trimmed with Brussels lace (B) draperies trimmed with Brussels lace (C) trimmed with Brussels lace were draperies (D) the draperies were trimmed with Brussels lace (E) draperies trimmed with Brussels lace were there ANSWERS AND EXPLANATIONS

Explanation for Correct Answer B : Choice (B) is correct. It avoids the error of the original by making the third item in the series a noun phrase, as the other two items are.

Explanation for Incorrect Answer A : Choice (A) involves an error in parallelism. The third item in the series should be a noun phrase like the other two ("a drawing room with a piano," "bath fixtures of silver"), not an independent clause. Explanation for Incorrect Answer C : Choice (C) involves an error in parallelism. The third item in the series should be a noun phrase like the other two ("a drawing room with a piano," "bath fixtures of silver"), not an independent clause. Explanation for Incorrect Answer D : Choice (D) involves an error in parallelism. The third item in the series should be a noun phrase like the other two ("a drawing room with a piano," "bath fixtures of silver"), not an independent clause. Explanation for Incorrect Answer E : Choice (E) involves an error in parallelism. The third item in the series should be a noun phrase like the other two ("a drawing room with a piano," "bath fixtures of silver"), not an independent clause. 2 Samuel Adams was by no means the first American to espouse the democratic cause, but he has been the first who conceived the party machinery that made it practical. (A) has been the first who conceived (B) had been the first who conceived (C) was the first having conceived (D) was the first to conceive (E) having been the first to conceive ANSWERS AND EXPLANATIONS

Explanation for Correct Answer D : Choice (D) is correct. It avoids the error of the original by using the past tense consistently. Explanation for Incorrect Answer A : Choice (A) involves an error in verb tense. It shifts from past tense ("was") to present perfect tense ("has been") for no reason. Explanation for Incorrect Answer B : Choice (B) involves an error in verb tense. It shifts from past tense ("was") to present perfect tense ("had been") for no reason. Explanation for Incorrect Answer C : Choice (C) involves an error in verb form. The participle "Having conceived" should be the infinitive form "to conceive." Explanation for Incorrect Answer E : Choice (E) creates a fragment. There is no subject for the verb "having been." 3 The plans were made too hastily, without enough thought behind it. (A) too hastily, without enough thought behind it (B) too hasty, without enough thought behind it (C) too hastily, without enough thought behind them (D) too hasty, and there is not enough thought behind them (E) too hastily, and there is not enough thought behind it ANSWERS AND EXPLANATIONS

Explanation for Correct Answer C :

Choice (C) is correct. It avoids the error of the original by using a plural pronoun ("them") to refer to the plural noun "plans." Explanation for Incorrect Answer A : Choice (A) involves an error in agreement. The singular pronoun "it" cannot correctly refer to the plural noun "plans." Explanation for Incorrect Answer B : Choice (B) involves improper modification. To modify the verb "made," "hasty" should be "hastily." Explanation for Incorrect Answer D : Choice (D) involves improper modification. To modify the verb "made," "hasty" should be "hastily." Explanation for Incorrect Answer E : Choice (E) involves an error in agreement. The singular pronoun "it" cannot correctly refer to the plural noun "plans." 4 Many psychologists do not use hypnosis in their practices, it is because they know very little about it and are wary of it as a result. (A) practices, it is because they know very little about it and are wary of it as a result. (B) practices because they know very little about it and are therefore wary of it. (C) practices for the reason that they know very little about it, with resulting wariness (D) practices because of knowing very little about it and therefore they are wary of it (E) practices, their knowledge of it being very little results in wariness of it ANSWERS AND EXPLANATIONS

Explanation for Correct Answer B : Choice (B) is correct. It avoids the error of the original by using a second verb phrase ("are therefore wary of it as a result") instead of a second independent clause. Explanation for Incorrect Answer A : Choice (A) involves improper coordination. Two complete thoughts ("Many...practices" and "it...result") are joined by only a comma. Explanation for Incorrect Answer C : Choice (C) involves improper phrasing. Whose "wariness" is being referred to is not clear. Explanation for Incorrect Answer D : Choice (D) involves improper coordination. Two complete thoughts ("Many...it" and" therefore...result") are joined only by a conjuction ("and"), without a comma to proceed it. Explanation for Incorrect Answer E : Choice (E) involves improper pronoun case. "It" should be "its." 5 No two of the specimens was sufficiently alike to warrant them being called members of a single species. (A) was sufficiently alike to warrant them being called (B) was sufficiently alike to warrant the calling of them (C) was sufficiently alike to warrant their being called (D) were sufficiently alike to warrant the calling of them (E) were sufficiently alike to warrant calling them ANSWERS AND EXPLANATIONS

Explanation for Correct Answer E : Choice (E) is correct. It avoids the error of the original by using a plural verb ("were") for the subject "two." Explanation for Incorrect Answer A : Choice (A) involves an error in agreement. The subject "two" requires a plural verb ("were"). Explanation for Incorrect Answer B : Choice (B) involves an error in agreement. The subject "two" requires a plural verb ("were"). Explanation for Incorrect Answer C : Choice (C) involves an error in agreement. The subject "two" requires a plural verb ("were"). Explanation for Incorrect Answer D : Choice (D) involves wordiness. The wordy phrase "the calling of them" should simply be "calling them." 6 My grandson thinks he can cook better than any other person at the fair; and he has the blue ribbons to prove it. (A) My grandson thinks he can cook better than any other person at the fair; and he (B) My grandson thinks he can cook better than any person at the fair, and he (C) My grandson thinks he can cook better than any person at the fair, consequently he (D) To think he can cook better than any other person at the fair, my grandson

(E) Thinking he can cook better than any other person at the fair, my grandson ANSWERS AND EXPLANATIONS

Explanation for Correct Answer B : Choice (B) is correct. It avoids the error of the original by using a comma to link two independent clauses joined by the conjunction "and." Explanation for Incorrect Answer A : Choice (A) involves improper coordination. It uses a semicolon where a comma is necessary. Explanation for Incorrect Answer C : Choice (C) creates an illogical sentence. It illogically suggests that the grandson's having blue ribbons is a result of his thinking that he is the best cook at the fair. Explanation for Incorrect Answer D : Choice (D) results in an illogical sentence. There is no relationship between the idea in the first part of the sentence ("To think . . . fair") and the idea in the last part of the sentence ("my grandson . . . to prove it."). Explanation for Incorrect Answer E : Choice (E) creates an illogical sentence. It does not make sense to say that the blue ribbons that the grandson had somehow prove that he was "thinking he can cook better . . . fair." 7 Differing only slightly from the Greeks were the Roman theaters, which were often freestanding rather than part of a hillside.

(A) Differing only slightly from the Greeks were the Roman theaters, which (B) Differing only slightly from Greek theaters, Roman theaters (C) Differing only in the slightest from the Greeks were the Roman theaters, which (D) The Greeks differed only slightly from the Romans, they (E) The Greek theaters differed from the Roman theaters only slightly, where they ANSWERS AND EXPLANATIONS

Explanation for Correct Answer B : Choice (B) is correct. It avoids the error of the original by correctly comparing "Greek theaters" to "Roman theaters." Explanation for Incorrect Answer A : Choice (A) involves an illogical comparison. It compares "the Greeks" with "the Roman theaters." Explanation for Incorrect Answer C : Choice (C) involves an illogical comparison. It compares "the Greeks" with "the Roman theaters." Explanation for Incorrect Answer D : Choice (D) involves improper coordination. Two complete thoughts ("The Greeks...Romans" and "they...hillside") are joined by only a comma. Explanation for Incorrect Answer E : Choice (E) involves an error in pronoun reference. There is no place to which "where" can refer. 8

When chronological order is followed too mechanically, they are obscuring rather than clarifying important relationships. (A) When chronological order is followed too mechanically, they are obscuring rather than clarifying important relationships. (B) When chronological order is followed too mechanically, it obscures rather than clarifying important relationships. (C) Chronological order, if too mechanically followed, obscures rather than it clarifies important relationships. (D) Chronological order, if followed too mechanically, obscures rather than clarifies important relationships. (E) If you follow a too mechanical chronological order, it obscures rather than clarifies important relationships. ANSWERS AND EXPLANATIONS

Explanation for Correct Answer D : Choice (D) is correct. It avoids the error of the original by omitting the plural pronoun ("they"), which did not agree with the singular noun "chronological order." Explanation for Incorrect Answer A : Choice (A) involves an error in agreement. The plural pronoun "they" is used incorrectly to refer to the singular noun "chronological order." Explanation for Incorrect Answer B : Choice (B) involves an error in verb form. The verb "clarifying" should be "clarifies." Explanation for Incorrect Answer C : Choice (C) involves a pronoun error. The pronoun "it" is unnecessary. Explanation for Incorrect Answer E :

Choice (E) involves a pronoun error. It is not clear what the pronoun "it" is meant to refer to. 9 Small marine crustaceans known as krill are often fed to farm animals, but there is not much human consumption. (A) animals, but there is not much human consumption (B) animals, but consumption is not done much by people (C) animals but are rarely eaten by people (D) animals, but eating them is rarely done by humans (E) animals, but among people there is not much consumption ANSWERS AND EXPLANATIONS

Explanation for Correct Answer C : Choice (C) is correct. It avoids the error of the original by avoiding unclear pronoun usage. Explanation for Incorrect Answer A : Choice (A) involves an error in pronoun reference. "There" does not refer to anything in the sentence. Explanation for Incorrect Answer B : Choice (B) involves wordiness. The wordy phrase "consumption is not done much by people" should be simply "are rarely consumed by people" or "are rarely eaten by people." Explanation for Incorrect Answer D : Choice (D) involves an error in pronoun reference. The pronoun "them" incorrectly refers to the closest previous plural noun, "farm animals." Explanation for Incorrect Answer E :

Choice (E) involves an unclear reference. The object of "consumption" is not clear. 10 The educators remarks stressed that well-funded literacy programs are needed if everyone is to gain the skills required for survival in society. (A) that well-funded literacy programs are needed if everyone is to gain (B) that well-funded literacy programs needed in gaining (C) there is a need of well-funded literacy programs for everyone will gain (D) a need for well-funded literacy programs and everyone will gain (E) why well-funded literacy programs being necessary for everyone in gaining ANSWERS AND EXPLANATIONS

Explanation for Correct Answer A : Choice (A) is correct. It correctly uses a relative clause (introduced by "that") to indicate what the educator was stressing. Explanation for Incorrect Answer B : Choice (B) involves improper verb form. The verb "needed" lacks an auxillary verb ("are"). Explanation for Incorrect Answer C : Choice (C) involves improper coordination. Two complete thoughts ("The educator's...programs" and "everyone...society") are joined by only a conjunction ("for"), without the comma that should precede it. Explanation for Incorrect Answer D : Choice (D) involves improper coordination. Two complete thoughts ("The educator's...programs" and "everyone...society") are joined by only a conjunction ("and"), without the comma that should precede it.

Explanation for Incorrect Answer E : Choice (E) creates an illogical sentence. A noun can be "stressed," but an adverb ("why") cannot. 11 The Portuguese musical tradition known as fado, or fate, has been called the Portuguese blues because of their songs that bemoan someones misfortune, especially the loss of romantic love. (A) of their songs that bemoan someones (B) of their songs bemoaning their (C) its songs bemoan (D) the songs that bemoaned (E) of how it bemoans their ANSWERS AND EXPLANATIONS

Explanation for Correct Answer C : Choice (C) is correct. It avoids the error of the original by using the pronoun "its," which correctly refers to the noun "The Portuguese...fado." Explanation for Incorrect Answer A : Choice (A) involves an error in pronoun reference. The pronoun "their" does not refer to anything that comes before it. Explanation for Incorrect Answer B : Choice (B) involves an error in pronoun reference. The pronoun "their" does not refer to anything that comes before it. Explanation for Incorrect Answer D : Choice (D) creates a fragment. The subordinate clause "that...love" is not completed.

Explanation for Incorrect Answer E : Choice (E) involves an error in pronoun reference. The pronoun "their" does not refer to anything that comes before it. 12 Every year, toy manufacturers gather groups of children into playrooms, A observing their choices of toys as predicting which new products will B C become the most popular. No error D E ANSWERS AND EXPLANATIONS

Corrected Sentence: Every year, toy manufacturers gather groups of children into playrooms, oberving their choices of toys and predicting which new products will become the most popular. Explanation for Correct Answer B : The error in this sentence occurs at (B), where there is an incomplete verb form. The word "as" cannot properly join the verbs "observing" and "predicting" to form the compound predicate. Instead, the conjunction "and" should be used. Explanation for Incorrect Answer A : There is no error at (A). The verb "gather" appropriately agrees with its subject, "toy manufacturers." Explanation for Incorrect Answer C : There is no error at (C). The relative pronoun "which" appropriately refers to the noun "new products" in describing the kinds of products that "will become most popular." Explanation for Incorrect Answer D :

There is no error at (D). The adverb "most" properly modifies the adjective "popular." Explanation for Incorrect Answer E : There is an error in the sentence. 13 During the last fifty years, we come to take radio communication for A B C granted, but the mere suggestion that we could communicate in such a D fashion must once have seemed outlandish. No error E ANSWERS AND EXPLANATIONS Corrected Sentence: During the last fifty years, we have come to take radio communication for granted, but the mere suggestion that we could communicate in such a fashion must once have seemed outlandish. Explanation for Correct Answer B : The error in this sentence occurs at (B), where there is an improper verb tense. The introductory prepositional phrase "During the last fifty years" establishes that the action of the main clause (taking radio communications for granted) takes place over the whole period. Therefore, the present-tense verb, "come" should be in the present perfect tense ("have come"). Explanation for Incorrect Answer A : There is no error at (A). The preposition "During" is an appropriate and necessary part of the prepositional phrase "During the last fifty years." Explanation for Incorrect Answer C : There is no error at (C). The infinitive "to take" appropriately introduces its object, "radio communications." Explanation for Incorrect Answer D :

There is no error at (D). The preposition "in" appropriately introduces the prepositional phrase "in such a fashion," and the adjective "such" appropriately modifies "a fashion." Explanation for Incorrect Answer E : There is an error in the sentence. 14 The uncompromising tone of a recent city hall ordinance concerning the A B blocking of emergency vehicles in traffic jams carry a stern warning to C D motorists. No error E ANSWERS AND EXPLANATIONS Corrected Sentence: The uncompromising tone of a recent city call ordinance concerning the blocking of emergency vehicles in traffic jams carries a stern warning to motorists. Explanation for Correct Answer C : The error in this sentence occurs at (C), where there is subject-verb disagreement. The plural verb "carry" cannot refer to the singular subject, "The uncompromising tone." The singular verb "carries" is needed. Explanation for Incorrect Answer A : There is no error at (A). The adjective "uncompromising" appropriately modifies the noun "tone." Explanation for Incorrect Answer B : There is no error at (B). The participle "concerning" appropriately modifies the preceding noun "ordinance." Explanation for Incorrect Answer D : There is no error at (D). The preposition "to" properly links the noun phrase "a stern warning" to the noun "motorists."

Explanation for Incorrect Answer E : There is an error in the sentence. 15 Formed by volcanic eruptions over the last five million years, the Hawaiian A Islands containing an incredibly wide variety of speciesmany found B C nowhere else on Earth. No error D E ANSWERS AND EXPLANATIONS Corrected Sentence: Formed by volcanic eruptions over the last five million years, the Hawaiian Islands contain an incredibly wide variety of species-many found nowhere else on Earth. Explanation for Correct Answer B : The error in this sentence occurs at (B), where a sentence fragment is created. The use of the participle "containing," instead of the present-tense "contain," leaves the sentence without a main verb. Explanation for Incorrect Answer A : There is no error at (A). The preposition "over" properly introduces the prepositional phrase "over the last five million years," which establishes when the volcanic eruptions took place. Explanation for Incorrect Answer C : There is no error at (C). The adverb "incredibly" appropriately modifies the adjective "wide." Explanation for Incorrect Answer D : There is no error at (D). The phrase "nowhere else" properly indicates that the "wide variety of species" are found mainly in one place. Explanation for Incorrect Answer E : There is an error in the sentence.

16 Because the owl is usually nocturnal plus being virtually noiseless in flight, A B it is seldom seen by the casual observer. No error C D E ANSWERS AND EXPLANATIONS Corrected Sentence: Because the owl is usually nocturnal and is virtually noiseless in flight, it is seldom seen by the casual observer. Explanation for Correct Answer B : The error in this sentence occurs at (B), where there is an awkward construction. The phrase "and is" should be used in place of "plus being" to properly join one characteristic (nocturnal behavior) with the other (noiselessness in flight). Explanation for Incorrect Answer A : There is no error at (A). The singular verb, "is," agrees with its singular subject, "the owl," and the adverb "usually" properly modifies the adjective "nocturnal." Explanation for Incorrect Answer C : There is no error at (C). The pronoun "it" refers correctly to the noun "the owl." Explanation for Incorrect Answer D : There is no error at (D). The singular verb "is" agrees with its singular subject ("the owl"), and the adverb "seldom" appropriately modifies the verb "seen." Explanation for Incorrect Answer E : There is an error in the sentence.

17 An economical and efficient recycling center is accessible to the public, A B responsive to community needs, and comply with current federal regulations C governing waste disposal. No error D E ANSWERS AND EXPLANATIONS

Corrected Sentence: An economical and efficient recycling center is accessible to the public, responsive to community needs, and complies with current federal regulations governing waste disposal. Explanation for Correct Answer C : The error in this sentence occurs at (C), where there is subject-verb disagreement. The plural verb "comply" does not agree with its singular subject ("recycling center"). Explanation for Incorrect Answer A : There is no error at (A). The compound adjective, "economical and efficient," appropriately modifies "recycling center." Explanation for Incorrect Answer B : There is no error at (B). The singular verb "is" agrees with its singular subject, "recycling center," and the adjective "accessible" appropriately modifies "recycling center" to indicate one aspect (accessibility) of an "economical and efficient" recycling center. Explanation for Incorrect Answer D : There is no error at (D). The participle "governing" appropriately indicates the kind of regulations being discussed (those governing waste disposal). Explanation for Incorrect Answer E : There is an error in the sentence.

18 Jean Toomer was not only the author of Cane, a novel whose publication has A been viewed as marking the beginning of the Harlem Renaissance, but also a B C respected advisor among Quakers. No error D E ANSWERS AND EXPLANATIONS Corrected Sentence: The sentence contains no error. Explanation for Correct Answer E : There is no error in this sentence. Explanation for Incorrect Answer A : There is no error at (A). The phrase "not only" operates appropriately as the first part of the correlative construction "not only . . . but also." Explanation for Incorrect Answer B : There is no error at (B). The tense of the verb correctly indicates that the action described (the way the novel is viewed) is still going on. Explanation for Incorrect Answer C : There is no error at (C). The adverb "as" joins with the participle "marking" to produce an appropriate idiom. Explanation for Incorrect Answer D : There is no error at (D). The preposition "among" joins with the plural noun "Quakers" to correctly describe where Toomer was "a respected advisor." 19 Election results came in from upstate New York quite rapid, but the results A B from New York City were known even faster. No error C D E

ANSWERS AND EXPLANATIONS Corrected Sentence: Election results came in from upstate New York quite rapidly, but the results from New York City were known even faster. Explanation for Correct Answer B : The error in this sentence occurs at (B), where an adjective ("rapid") is incorrectly used to modify the verb "came." The adverb "rapidly" is needed. Explanation for Incorrect Answer A : There is no error at (A). The preposition "in" appropriately modifies the verb "came" to indicate where the election results came from. Explanation for Incorrect Answer C : There is no error at (C). The past tense of the verb is consistent with the tense previously established by the past-tense verb "came." Explanation for Incorrect Answer D : There is no error at (D). The adverb "even" appropriately modifies the adjective "faster" to indicate how quickly the results were known. Explanation for Incorrect Answer E : There is an error in the sentence. 20 As we rely more and more on the Internet, your need for effective security A B planning and design to safeguard data has increased. No error C D E ANSWERS AND EXPLANATIONS Corrected Sentence: As we rely more and more on the Internet, the need for effective security planning and design to safeguard data has increased. Explanation for Correct Answer B :

The error in this sentence occurs at (B), where there is an error in pronoun use. There is nothing in the sentence to which the possessive pronoun "your" can logically refer. Explanation for Incorrect Answer A : There is no error at (A). The subordinating conjuntion "As" appropriately introduces the dependent clause ("As we rely . . . on the Internet"). Explanation for Incorrect Answer C : There is no error at (C). The infinitive "to safeguard" appropriately indicates the purpose of "effective . . . design" (to safeguard data). Explanation for Incorrect Answer D : There is no error at (D). The tense of the verb "has increased" appropriately indicates that the action described (the increasing need for effective planning) is ongoing. Explanation for Incorrect Answer E : There is an error in the sentence. 21 The book is essentially a detailed and very well documented record of what A B C happened to each of the protestors. No error D E ANSWERS AND EXPLANATIONS Corrected Sentence: Explanation for Correct Answer E : There is no error in this sentence. Explanation for Incorrect Answer A : There is no error at (A). The singular verb "is" agrees with its singular subject "the book," and the adverb "essentially" appropriately modifies the verb "is."

Explanation for Incorrect Answer B : There is no error at (B). The adverbs "very" and "well" combine to appropriately modify the adjective "documented." Explanation for Incorrect Answer C : There is no error at (C). The preposition "of" combines with the relative pronoun "what" to produce an appropriate idiom indicating what was documented in the book. Explanation for Incorrect Answer D : There is no error at (D). The pronoun "each" combines with the preposition "of" to form an appropriate idiom linking "what happened" to the group to which it happened. 22 Experts agree that permanently modifying eating and exercise habits rather A than merely dieting for brief periods are the key to controlling weight. B C D No error E ANSWERS AND EXPLANATIONS Corrected Sentence: Experts agree that permanently modifying eating and exercise habits rather than merely dieting for brief periods is the key to controlling weight. Explanation for Correct Answer C : The error in this sentence occurs at (C), where there is an incorrect verb form. The plural verb "are" should be the singular "is" to agree with the singular gerund, "modifying eating and exercise habits." Explanation for Incorrect Answer A :

There is no error at (A). The plural verb ("agree") agrees with its subject, "Experts," and the relative pronoun "that" appropriately introduces the phrase that immediately follows. Explanation for Incorrect Answer B : There is no error at (B). The gerund "dieting" is appropriately modified by the adverb "merely." Explanation for Incorrect Answer D : There is no error at (D). The preposition "to" properly links the noun "the key" with the gerund "controlling weight." Explanation for Incorrect Answer E : There is an error in the sentence. 23 The ability to control the plots of our dreams is a skill, researchers have A B shown, that we can learn if you want to change recurrent dreams. No error C D E ANSWERS AND EXPLANATIONS Corrected Sentence: The ability to control the plots of our dreams is a skill, researchers have shown, that we can learn if we want to change recurrent dreams. Explanation for Correct Answer D : The error in this sentence occurs at (D), where there is improper prounoun use. The second-person pronoun "you" should be changed to the first-person plural "we" to be consistent with the earlier use of "we." Explanation for Incorrect Answer A : There is no error at (A). The infinitive "to control" properly modifies the preceding noun, "The ability." Explanation for Incorrect Answer B :

There is no error at (B). The singular verb "is" agrees with its singular subject, "The ability." Explanation for Incorrect Answer C : There is no error at (C). The verb tense appropriately establishes that the action described (what researchers have determined about the ability to control dreams) has already taken place. Explanation for Incorrect Answer E : There is an error in the sentence. 24 In swimming as to soccer, Evangelina proved time after time to be an abler A B C competitor than Juanita. No error D E ANSWERS AND EXPLANATIONS

Corrected Sentence: In swimming as in soccer, Evangelina proved time after time to be an abler competitor than Juanita. Explanation for Correct Answer B : The error in this sentence occurs at (B), where there is an improper idiom. The preposition "in" should be used with the noun "soccer" to convey the idea that what happens "in swimming" is also what happens "in soccer." Explanation for Incorrect Answer A : There is no error at (A). The preposition "in" combines with the gerund "swimming" to create an appropriate idiom. Explanation for Incorrect Answer C : There is no error at (C). The comparative form of the adjective "able" is correct and properly modifies the noun "competitor."

Explanation for Incorrect Answer D : There is no error at (D). The conjunction "than" is used correctly to introduce the object of the comparison ("Juanita"). Explanation for Incorrect Answer E : There is an error in the sentence. 25 The common cold is one of our most indiscriminate diseases; it makes no A B distinction between you and me, millionaires and paupers, or athletes and C D couch potatoes. No error E ANSWERS AND EXPLANATIONS Corrected Sentence: Explanation for Correct Answer E : There is no error in this sentence. Explanation for Incorrect Answer A : There is no error at (A). The pronoun "one" is used correctly to refer to "The common cold," and the preposition "of" appropriately introduces the prepositional phrase that modifies "The common cold." Explanation for Incorrect Answer B : There is no error at (B). The pronoun "it" correctly refers to the noun "The common cold," and the singular verb "makes" agrees with its singular subject, "it." Explanation for Incorrect Answer C : There is no error at (C). The noun "distinction" links appropriately with the preposition "between" to create an appropriate idiom. Explanation for Incorrect Answer D :

There is no error at (D). Both pronouns ("you" and "me") are properly in the objective case. 26 Like his other cookbooks, in his new book Chef Louis offers lengthy A explanations of what he considers to be basic cooking principles. No error B C D E ANSWERS AND EXPLANATIONS Corrected Sentence: As he does in his other cookbooks, in his new book Chef Louis offers lengthy explanations of what he considers to be basic cooking principles. Explanation for Correct Answer A : The error in this sentence occurs at (A), where there is an illogical comparison. It would be appropriate to compare what is in "his other cookbooks" with what is in "his new book," but instead the sentence illogically compares a thing ("his other cookbooks") with a person ("Chef Louis"). Explanation for Incorrect Answer B : There is no error at (B). The preposition "of" combines with the pronoun "what" to create an appropriate idiom. Explanation for Incorrect Answer C : There is no error at (C). The pronoun "he" correctly refers to its antecedent "Chef Louis," and the singular verb "considers" agrees with its singular subject, "he." Explanation for Incorrect Answer D : There is no error at (D). The infinitive "to be" joins with the verb "considers" to appropriately indicate what the "lengthy explanations" are about.

Explanation for Incorrect Answer E : There is an error in the sentence. 27 Paul Ecke, flower grower and hybridizer, became known as Mr. Poinsettia A B after developing new varieties of the flower and by pioneering it as a living C D symbol of Christmas. No error E ANSWERS AND EXPLANATIONS

Corrected Sentence: Paul Ecke, flower grower and hybridizer, became known as "Mr. Poinsettia" after developing new varieties of the flower and pioneering it as a living symbol of Christmas. Explanation for Correct Answer C : The error in this sentence occurs at (C), where there are excess words. The preposition "by" is unnecessary. Explanation for Incorrect Answer A : There is no error at (A). The past-tense verb "became" appropriately establishes that Mr. Ecke was known a "Mr. Poinsettia" only after he developed "new varieties of the flower." Explanation for Incorrect Answer B : There is no error at (B). The adjective "known" appropriately modifies the subject of the sentence "Paul Ecke." Explanation for Incorrect Answer D : There is no error at (D). The prepositon "as" joins with the noun phrase "a living symbol" to create an appropriate idiom.

Explanation for Incorrect Answer E : There is an error in the sentence. 28 Long thought of as a quiet, stuffy place where people just borrowed books, A B C libraries have been changing their images dramatically over the last few D years. No error D ANSWERS AND EXPLANATIONS Corrected Sentence: Long thought of as quiet, stuffy places where people just borrowed books, libraries have been changing their images dramatically over the last few years. Explanation for Correct Answer B : The error in this sentence occurs at (B), where there is noun-noun disagreement. To agree with the plural "libraries," the singular noun "place" should be changed to the plural "places." Explanation for Incorrect Answer A : There is no error at (A). The verb "thought" combines with the preposition "of" to create an appropriate idiom. Explanation for Incorrect Answer C : There is no error at (C). The plural noun "people" is logical here because more than one library is being talked about. Explanation for Incorrect Answer D : There is no error at (D). The present perfect progressive tense of the verb appropriately indicates that the action described may be ongoing. Explanation for Incorrect Answer E : There is an error in the sentence.

29 To understand twentieth-century economic practices, we must be sufficiently A B familiar with Keynesian theories, whether one agrees with them or not. C D No error E ANSWERS AND EXPLANATIONS Corrected Sentence: To understand twentieth-century economic practices, one must be sufficiently familiar with Keynesian theories, whether one agrees with them or not. Explanation for Correct Answer B : The error in this sentence occurs at (B), where there is incorrect pronoun use. The first-person plural pronoun "we" is not consistent with the later pronoun "one." Explanation for Incorrect Answer A : There is no error at (A). The infinitive "To understand" appropriately indicates that a familiarity with Keynesian theories will facilitate one's understanding of "twentieth-century economic practices." Explanation for Incorrect Answer C : There is no error at (C). The adjective "familiar" joins with the preposition "with" to form an appropriate idiom. Explanation for Incorrect Answer D : There is no error at (D). In the prepositional phrase "with them," the object of the preposition, "them," must be in the objective case, as it is here. Explanation for Incorrect Answer E : There is an error in the sentence.

(1) Not many children leave elementary school and they have not heard of Pocahontas heroic rescue of John Smith from her own people, the Powhatans. (2) Generations of Americans have learned the story of a courageous Indian princess who threw herself between the Virginia colonist and the clubs raised to end his life. (3) The captive himself reported the incident. (4) According to that report, Pocahontas held his head in her arms and laid her own upon his to save him from death. (5) But can Smiths account be trusted? (6) Probably it cannot, say several historians interested in dispelling myths about Pocahontas. (7) According to these experts, in his eagerness to find patrons for future expeditions, Smith changed the facts in order to enhance his image. (8) Portraying himself as the object of a royal princess devotion may have merely been a good public relations ploy. (9) Research into Powhatan culture suggests that what Smith described as an execution might have been merely a ritual display of strength. (10) Smith may have been a character in a drama in which even Pocahontas was playing a role. (11) As ambassador from the Powhatans to the Jamestown settlers, Pocahontas headed off confrontations between mutually suspicious parties. (12) Later, after her marriage to colonist John Rolfe, Pocahontas traveled to England, where her diplomacy played a large part in gaining support for the Virginia Company. 30 What is the best way to deal with sentence 1 (reproduced below) ? Not many children leave elementary school and they have not heard of Pocahontas heroic rescue of John Smith from her own people, the Powhatans.

(A) Leave it as it is (B) Switch its position with that of sentence 2. (C) Change leave to have left. (D) Change and they have not heard to without having heard. (E) Remove the comma and insert known as the. ANSWERS AND EXPLANATIONS

Explanation for Correct Answer D : Choice (D) is correct. It properly explains that most children hear the Pocahontas story before they leave elementary school. Explanation for Incorrect Answer A : Choice (A) is unsatisfactory because the original sentence connects the two main ideas--children leaving school and the Pocahontas story--with only the conjunction "and." The sentence thus offers no clue about the relationship between the two ideas. Explanation for Incorrect Answer B : Choice (B) is unsatisfactory because it is logical to give the names of the principal figures in a story or event before telling the story, not after-especially when the names are familiar. Explanation for Incorrect Answer C : Choice (C) is unsatisfactory because it repeats the error of the original in failing to explain the relationship between the children and the story. Explanation for Incorrect Answer E : Choice (E) is unsatisfactory because the original correctly refers to Pocahontas's tribe. 31 In context, which of the following is the best way to revise the underlined wording in order to combine sentences 3 and 4 (reproduced below)? The captive himself reported the incident. According to that report Pocahontas held his head in her arms and laid her own upon his to save him from death.

(A) The captive himself reported the incident, according to which (B) Since then, the captive reported the incident, which said that (C) Consequently, the captive himself reports that (D) It seems that in the captives report of the incident he says that (E) According to the captives own report of the incident, ANSWERS AND EXPLANATIONS

Explanation for Correct Answer E : Choice (E) is correct. The resulting sentence maintains the sense of the original while eliminating the redundancy. Explanation for Incorrect Answer A : Choice (A) is unsatisfactory because it contains an unclear referent: "which" seems to refer to the incident itself rather than to the report. Explanation for Incorrect Answer B : Choice (B) is unsatisfactory because the word "which" appears to refer to the incident, when it can logically refer only to the report of the incident. Explanation for Incorrect Answer C : Choice (C) is unsatisfactory because the word "consequently" suggests incorrectly that Smith's report is a consequence of the legend. Explanation for Incorrect Answer D : Choice (D) is unsatisfactory because it uses the unnecessary phrase "it seems" to relate a fact. 32 Which of the following phrases is the best to insert at the beginning of sentence 10 to link it to sentence 9?

(A) Far from being in mortal danger, (B) If what he says is credible, (C) What grade school history never told you is this: (D) They were just performing a ritual, and (E) But quite to the contrary, ANSWERS AND EXPLANATIONS

Explanation for Correct Answer A : Choice (A) is correct. It links sentence 10 to the rest of the paragraph by explaining the harmlessness of the "ritual display" mentioned in the previous sentence (and thus clarifies the contrast between Smith's account and the probable facts). Explanation for Incorrect Answer B : Choice (B) is unsatisfactory because sentence 10 outlines a scenario that challenges Smith's "life-or-death" account, implying that Smith is not a credible source. Explanation for Incorrect Answer C : Choice (C) is unsatisfactory because the inserted phrase unhelpfully interrupts the connection between the "ritual display" introduced in sentence 9 and the explanation of it in sentence 10. Explanation for Incorrect Answer D : Choice (D) is unsatisfactory because the use of "and" implies that the "ritual" and the "drama" are two different events, whereas the "drama" actually refers to the "ritual display." Explanation for Incorrect Answer E :

Choice (E) is unsatisfactory because nothing in sentence 10 is contrary to sentence 9; the latter sentence logically follows the former. 33 Which of the following best describes the relationship between sentences 9 and 10?

(A) Sentence 10 concludes that the theory mentioned in sentence 9 is wrong. (B) Sentence 10 adds to information reported in sentence 9. (C) Sentence 10 provides an example to illustrate an idea presented in sentence 9. (D) Sentence 10 poses an argument that contradicts the point made in sentence 9. (E) Sentence 10 introduces a new source that confirms the claims made in sentence 9. ANSWERS AND EXPLANATIONS

Explanation for Correct Answer B : Choice (B) is correct. Sentence 10 elaborates on the information about what may have really happened to Smith presented in sentence 9. Explanation for Incorrect Answer A : Choice (A) is unsatisfactory because sentence 10 offers only support for the claim made in sentence 9. Explanation for Incorrect Answer C : Choice (C) is unsatisfactory because the information in sentence 10 is not an "example"; rather, it is a reasoned clarification of what may have happened to Smith. Explanation for Incorrect Answer D :

Choice (D) is unsatisfactory because nothing about sentence 10 contradicts sentence 9. Explanation for Incorrect Answer E : Choice (E) is unsatisfactory because sentence 10 does not make use of any new sources. 34 Which of the following would be the best to insert before sentence 11 to introduce the third paragraph?

(A) It is crucial to consider the political successes as well as the shortcomings of Pocahontas. (B) The Pocahontas of legend is the most interesting, but the historical Pocahontas is more believable. (C) If legend has overemphasized the bravery of Pocahontas, it has underplayed her political talents. (D) To really know Pocahontas, we must get beyond myth and legend to the real facts about her private life. (E) Perhaps we will never really know the real Pocahontas. ANSWERS AND EXPLANATIONS

Explanation for Correct Answer C : Choice (C) is correct. The third paragraph gives two detailed examples of Pocahontas's political successes in later life. Explanation for Incorrect Answer A : Choice (A) is unsatisfactory because the passage does not mention any of Pocahontas's shortcomings. Explanation for Incorrect Answer B :

Choice (B) is unsatisfactory because focusing on the believability of historical facts is odd and unnecessary. Explanation for Incorrect Answer D : Choice (D) is unsatisfactory because the information in paragraph 3 deals primarily with Pocahontas's public life, not her private life. Explanation for Incorrect Answer E : Choice (E) is unsatisfactory because the third paragraph gives detailed information about Pocahontas that is not in dispute. 35 What information is most logical to add immediately after sentence 12 ?

(A) How Rolfe and Pocahontas happened to meet and marry (B) Details about other versions of the legend concerning John Smith (C) Reasons for the confrontations between the Powhatans and the Jamestown settlers (D) An account of Rolfes life and work in Virginia (E) A brief summary of the other public events in Pocahontas life ANSWERS AND EXPLANATIONS

Explanation for Correct Answer E : Choice (E) is correct. This information is consistent with the information presented in the first two sentences of the paragraph. Explanation for Incorrect Answer A : Choice (A) is unsatisfactory because the first two sentences of the paragraph present events chronologically, and it would be illogical to describe

Pocahontas's marriage, which occurred before the events described in sentence 12, at the end of the paragraph Explanation for Incorrect Answer B : Choice (B) is unsatisfactory because the passage has moved from a discussion of Smith's account to a discussion of Pocahontas's life; to return to Smith at this point would be illogical. Explanation for Incorrect Answer C : Choice (C) is unsatisfactory because this information would need to be presented directly after its introduction in sentence 11. Explanation for Incorrect Answer D : Choice (D) is unsatisfactory because the focus of the paragraph is Pocahontas, not her husband.

Section 6: Math Section 7: Critical Reading


1 The success of Notes of a Native Son ------- author James Baldwin as one of the most ------- essayists of his time. (A) buoyed . . irrelevant (B) established . . prominent (C) surrendered . . prolific (D) decried . . cynical (E) categorized . . mundane ANSWERS AND EXPLANATIONS

Explanation for Correct Answer B :

Choice (B) is correct. Establish means to cause to be recognized and accepted. Prominent means widely and popularly known. If one were to insert these terms into the text, the sentence would read The success of Notes of a Native Son established author James Baldwin as one of the most prominent essayists of his time. The success of Notes of a Native Son gave Baldwin recognition and acceptance within literary circles. Explanation for Incorrect Answer A : Choice (A) is incorrect. Buoyed means supported or uplifted. Irrelevant means unrelated or insignificant. If one were to insert these terms into the text, the sentence would read The success of Notes of a Native Son buoyed author James Baldwin as one of the most irrelevant essayists of his time. Success can enhance a writers reputation, but it is illogical to claim that it would cause Baldwin to become insignificant. Explanation for Incorrect Answer C : Choice (C) is incorrect. Surrendered means abandoned. Prolific means to produce abundant works or results. If one were to insert these terms into the text, the sentence would read The success of Notes of a Native Son surrendered author James Baldwin as one of the most prolific essayists of his time. Many successful authors are prolific, but it does not make sense to say that success abandoned Baldwin as a prolific essayist. Explanation for Incorrect Answer D : Choice (D) is incorrect. Decried means denounced or belittled. Cynical means negative or pessimistic. If one were to insert these terms into the text, the sentence would read The success of Notes of a Native Son decried author James Baldwin as one of the most cynical essayists of his time. Some critics may have criticized Baldwin for being cynical, but a book cannot denounce its author. Explanation for Incorrect Answer E : Choice (E) is incorrect. Categorized means classified, and mundane means ordinary. If one were to insert these terms into the text, the sentence would read The success of Notes of a Native Son categorized author James

Baldwin as one of the most mundane essayists of his time. It is unlikely that success would classify or label an author as ordinary. 2 In many parts of the world, people use rice as a central rather than a ------part of their daily diets. (A) pivotal (B) ritualistic (C) salient (D) supplementary (E) solemn ANSWERS AND EXPLANATIONS

Explanation for Correct Answer D : Choice (D) is correct. "Supplementary" means something added or in addition to. If one were to insert this term into the text, the sentence would read "In many parts of the world, people use rice as a central rather than a supplementary part of their daily diets." The phrase "rather than" indicates that the blanked word will mean the opposite of the word "central." In this context, "supplementary" does indeed have the opposite meaning of "central." Explanation for Incorrect Answer A : Choice (A) is incorrect. "Pivotal" means being of vital or significant importance. If one were to insert this term into the text, the sentence would read "In many parts of the world, people use rice as a central rather than a pivotal part of their daily diets." "Pivotal" and "central" are synonymous, thus it makes little sense to claim that people use rice as a central rather than a "pivotal" part of their diet. Explanation for Incorrect Answer B : Choice (B) is incorrect. "Ritualistic" means advocating or practicing ritual. If one were to insert this term into the text, the sentence would read "In

many parts of the world, people use rice as a central rather than a ritualistic part of their daily diets." A "ritualistic," or ceremonial, use of rice does not necessarily mean that the rice is not a "central" part of the diet. Explanation for Incorrect Answer C : Choice (C) is incorrect. "Salient" means prominent. If one were to insert this term into the text, the sentence would read "In many parts of the world, people use rice as a central rather than a salient part of their daily diets." A "salient" part of a diet would be an important part. It is illogical to contrast the use of rice as a "central" part of the diet with the use of rice as an important part of the diet. Explanation for Incorrect Answer E : Choice (E) is incorrect. "Solemn" means somberly or gravely impressive. If one were to insert this term into the text, the sentence would read "In many parts of the world, people use rice as a central rather than a solemn part of their daily diets." It makes little sense to describe rice as a "solemn," or somber, part of a diet. 3 Victor gained a reputation for being a ------- because he constantly bullied other children. (A) bungler (B) ruffian (C) stickler (D) daredevil (E) naysayer ANSWERS AND EXPLANATIONS

Explanation for Correct Answer B : Choice (B) is correct. "Ruffian" describes a person who is a hoodlum or bully. If one were to insert this term into the text, the sentence would read "Victor gained a reputation for being a ruffian because he constantly bullied

other children." Bullying other children is precisely the behavior that would give Victor a reputation as a ruffian. Explanation for Incorrect Answer A : Choice (A) is incorrect. "Bungler" describes a person who is clumsy or inept in behavior. If one were to insert this term into the text, the sentence would read "Victor gained a reputation for being a bungler because he constantly bullied other children." The claim that Victor had a reputation for being a "bungler" does not fit with the claim that "he bullied other children," as a bully is not necessarily clumsy. Explanation for Incorrect Answer C : Choice (C) is incorrect. "Stickler" describes a person who insists on something unyieldingly. If one were to insert this term into the text, the sentence would read "Victor gained a reputation for being a stickler because he constantly bullied other children." Although Victor may be a "stickler," or perfectionist, his reputation is based on his tendency to bully other children. Explanation for Incorrect Answer D : Choice (D) is incorrect. "Daredevil" describes a person who is recklessly bold. If one were to insert this term into the text, the sentence would read "Victor gained a reputation for being a daredevil because he constantly bullied other children." While Victor may be a "daredevil," or risktaker, the sentence offers no evidence of this. Explanation for Incorrect Answer E : Choice (E) is incorrect. "Naysayer" describes a person who opposes in order to take a pessimistic view. If one were to insert this term into the text, the sentence would read "Victor gained a reputation for being a naysayer because he constantly bullied other children." The sentence offers no evidence that Victor is a "naysayer," or contrarian. 4

Paradoxically, the senator was both a ------- and -------: she publicly defended the rights and wisdom of the people, but she often spoke with a disdainful air of superiority. (A) demagogue . . a maverick (B) conservative . . an anarchist (C) populist . . an elitist (D) moderate . . a reactionary (E) partisan . . a snob ANSWERS AND EXPLANATIONS

Explanation for Correct Answer C : Choice (C) is correct. A "populist" is an advocate for the interests and rights of the common people. An "elitist" believes that certain persons deserve favored treatment by virtue of their perceived superiority. If one were to insert these terms into the text, the sentence would read "Paradoxically, the senator was both a populist and an elitist: she publicly defended the rights and wisdom of the people, but she often spoke with a disdainful air of superiority." The term "paradoxically" indicates that the two missing terms will be contradictory. The senator's behavior is inconsistent because she supports common people publically, while believing in the superiority of certain classes of people. Explanation for Incorrect Answer A : Choice (A) is incorrect. A "demagogue" is a leader who obtains power by means of impassioned appeals to the emotions and prejudices of the populace. A "maverick" is one that resists adherence to a group. If one were to insert these terms into the text, the sentence would read "Paradoxically, the senator was both a demagogue and a maverick: she publicly defended the rights and wisdom of the people, but she often spoke with a disdainful air of superiority." A "demagogue," a person who stirs up emotion, can also be a "maverick" or independent politician. The two are not necessarily opposed to one another, and so the behavior of a person who exhibits both of these characteristics would not logically be described as paradoxical. Explanation for Incorrect Answer B :

Choice (B) is incorrect. "Conservative" is one favoring traditional values and views. "Anarchist" is an advocate of political disorder and confusion. If one were to insert these terms into the text, the sentence would read "Paradoxically, the senator was both a conservative and an anarchist: she publicly defended the rights and wisdom of the people, but she often spoke with a disdainful air of superiority." A "conservative" politician does not necessarily defend the rights of the common people. An "anarchist" opposes the idea of government and is unlikely to hold a government office. Explanation for Incorrect Answer D : Choice (D) is incorrect. "Moderate" is one who holds average views or opinions. "Reactionary" means an opponent of progress or liberalism. If one were to insert these terms into the text, the sentence would read "Paradoxically, the senator was both a moderate and a reactionary: she publicly defended the rights and wisdom of the people, but she often spoke with a disdainful air of superiority." "Moderate," or middle-ground, and "reactionary" are not the polar opposites implied by the term "paradoxically." Explanation for Incorrect Answer E : Choice (E) is incorrect. "Partisan" is a supporter or proponent of a party, cause, faction, person, or idea. A "snob" looks down on people he or she regards as socially inferior. If one were to insert these terms into the text, the sentence would read "Paradoxically, the senator was both a partisan and snob: she publicly defended the rights and wisdom of the people, but she often spoke with a disdainful air of superiority." While a "snob" would speak with an air of superiority, it makes little sense to contrast a "snob" with a "partisan," or someone favoring a particular political party. Furthermore, a partisan would not necessarily defend the rights of the common people. 5 The geologist speculated that eons ago, before the area was -------, the present-day island was actually a hilltop in a vast forest. (A) inundated

(B) situated (C) rejuvenated (D) supplanted (E) excavated ANSWERS AND EXPLANATIONS

Explanation for Correct Answer A : Choice (A) is correct. "Inundated" means to cover completely as in a flood. If one were to insert this term into the text, the sentence would read "The geologist speculated that eons ago, before the area was inundated, the present-day island was actually a hilltop in a vast forest." The words "before" and "actually" in this sentence suggest that something happened to transform the hilltop into an island. An inundation of water would be the most logical cause of such a transformation. Explanation for Incorrect Answer B : Choice (B) is incorrect. "Situated" means having a location. If one were to insert this term into the text, the sentence would read "The geologist speculated that eons ago, before the area was situated, the present-day island was actually a hilltop in a vast forest." The location, or where the island is "situated," has not changed. Explanation for Incorrect Answer C : Choice (C) is incorrect. "Rejuvenated" means restored to an original or new condition. If one were to insert this term into the text, the sentence would read "The geologist speculated that eons ago, before the area was rejuvenated, the present-day island was actually a hilltop in a vast forest." While parts of the area may have been rejuvenated, or restored, this would not have turned the area from a hilltop into an island. Explanation for Incorrect Answer D : Choice (D) is incorrect. "Supplanted" means displaced and substituted for. If one were to insert this term into the text, the sentence would read "The geologist speculated that eons ago, before the area was supplanted, the

present-day island was actually a hilltop in a vast forest." The information in the sentence does not support the idea that the area was supplanted, or displaced. Explanation for Incorrect Answer E : Choice (E) is incorrect. "Excavated" means removed or exposed by digging. If one were to insert this term into the text, the sentence would read "The geologist speculated that eons ago, before the area was excavated, the present-day island was actually a hilltop in a vast forest." Although the area may have been excavated, or exhumed, this does not explain how the hilltop became an island. Passage 1 Any wildlife biologist can tell you how many deer a given area can supporthow much browse there is for the deer to eat before they begin to suppress the Line reproduction of trees, before they begin to starve in 5 the winter. Any biologist can calculate how many wolves a given area can support too, in part by counting the number of deer. And so on, up and down the food chain. Its not an exact science, but it comes pretty closeat least compared to figuring 10 out the carrying capacity of Earth for human beings, which is an art so dark that anyone with any sense stays away from it. Passage 2 Estimates of the number of humans that Earth can sustain have ranged in recent decades from fewer than a billion to more than a trillion. Such elasticity is probably unavoidable, since carrying capacity is essentially a subjective term. It makes little sense to talk about carrying capacity in relationship to humans, who are capable of adapting and altering both their culture and their physical

15

20

environment, and can thus defy any formula that might settle the matter. The number of people that Earth can support depends on how we on Earth want to live, on what we want to consume, and on what we regard as a crowd.

6 Both passages support which of the following conclusions about Earths carrying capacity for humans? (A) It is routinely underestimated by biologists. (B) It cannot be easily determined, given numerous variables and unknowns. (C) It has only recently become the subject of considerable scientific debate. (D) It is a valuable concept despite its apparent shortcomings. (E) It has increased as a result of recent technological innovations. ANSWERS AND EXPLANATIONS

Explanation for Correct Answer B : Choice (B) is correct. Both passages discuss the difficulty of accurately estimating how many people the Earth can support. Passage 1 asserts that estimating the Earth's "carrying capacity" is a "dark art," and Passage 2 explains why such estimates are problematic. Explanation for Incorrect Answer A : Choice (A) is incorrect. Passage 2 asserts that biologists' estimations of the Earth's "carrying capacity" for humans cover a broad range, but neither author claims that these existing "estimates" are too low. Explanation for Incorrect Answer C : Choice (C) is incorrect. Nothing in these passages suggests that the debate regarding human "carrying capacity" is a new one. Explanation for Incorrect Answer D :

Choice (D) is incorrect. Both passages clearly question the value of estimating the Earth's "carrying capacity" for humans. Passage 1 declares that "anyone with any sense stays away" from the concept, and Passage 2 says that "it makes little sense to talk about carrying capacity in relationship to humans." Explanation for Incorrect Answer E : Choice (E) is incorrect. The first passage does not discuss recent "technological innovations" at all, and the second passage supports this conclusion indirectly at best. 7 The author of Passage 1 refers to Any wildlife biologist in line 1 and Any biologist in line 5 to emphasize the point that (A) a particular type of calculation can be made with great confidence (B) scientific findings often meet with resistance from the general public (C) certain beliefs are rarely questioned by scientists (D) most biologists are concerned with issues related to wildlife mortality (E) all biologists must be skilled at applying mathematical formulas ANSWERS AND EXPLANATIONS

Explanation for Correct Answer A : Choice (A) is correct. The author of Passage 1 contrasts the ease of calculating how many animals an area can support with the difficulty of making such a calculation for people. The use of the word "any" conveys the author's belief that predicting an animal population's "carrying capacity" is a relatively simple task for biologists. Explanation for Incorrect Answer B : Choice (B) is incorrect. Passage 1 does not include any reference to the "general public."

Explanation for Incorrect Answer C : Choice (C) is incorrect. The author questions biologists' ability to accurately estimate the Earth's "carrying capacity" for humans but makes no reference to "beliefs" of any kind. Explanation for Incorrect Answer D : Choice (D) is incorrect. It may be true that the majority of biologists care about "wildlife mortality," but the author of Passage 1 makes no such claim. Explanation for Incorrect Answer E : Choice (E) is incorrect. While one might infer that the biologists mentioned in Passage 1 need to be skilled in mathematics, this is certainly not a point of emphasis in the passage. 8 Both authors would agree that the Estimates (Passage 2, line 13) are (A) overly generous (B) largely undocumented (C) often misunderstood (D) politically motivated (E) essentially unreliable ANSWERS AND EXPLANATIONS

Explanation for Correct Answer E : Choice (E) is correct. Passage 2 highlights the "elasticity" of scientists' "estimates," suggesting that such figures are fundamentally "unreliable." The author of Passage 1 echoes this assertion in describing the estimation of the Earth's "carrying capacity" for humans as an inexact science. Explanation for Incorrect Answer A :

Choice (A) is incorrect. Passage 2 asserts that biologists' estimations of the Earth's "carrying capacity" for humans cover a broad range, but neither author claims that these existing estimates are too high. Explanation for Incorrect Answer B : Choice (B) is incorrect. Neither author claims that the "estimates" in Passage 2 are generally "undocumented." Explanation for Incorrect Answer C : Choice (C) is incorrect. While the authors question the accuracy of certain carrying capacity "estimates," there is no suggestion that these figures have been "misunderstood." Explanation for Incorrect Answer D : Choice (D) is incorrect. Neither author ascribes a political motivation to the scientists responsible for the "estimates" mentioned in Passage 2. 9 Which of the following best describes the relationship between the two passages? (A) Passage 1 offers a hypothesis that is explicitly refuted in Passage 2. (B) Passage 1 describes a popular misconception that is exemplified by Passage 2. (C) Passage 2 presents an argument that elaborates on a point made in Passage 1. (D) Passage 2 defends a position that is attacked in Passage 1. (E) Passage 2 provides an anecdote that confirms the theory advanced in Passage 1. ANSWERS AND EXPLANATIONS

Explanation for Correct Answer C :

Choice (C) is correct. Passage 1 claims that estimating the Earth's "carrying capacity" for humans is "not an exact science." Passage 2 elaborates on this point by explaining how "carrying capacity" is a subjective concept when applied to humans. Explanation for Incorrect Answer A : Choice (A) is incorrect. Passage 2 supports, not refutes, the claim made in Passage 1 that scientists should stay away from "carrying capacity" estimates. Explanation for Incorrect Answer B : Choice (B) is incorrect. While the notion that human sustainability can be easily estimated may be a popular misunderstanding, Passage 2 counters, rather than exemplifies, such a "misconception." Explanation for Incorrect Answer D : Choice (D) is incorrect. Passage 1 criticizes scientists who attempt to calculate the Earth's "carrying capacity" for humans, but the author of Passage 2 agrees with this criticism. Explanation for Incorrect Answer E : Choice (E) is incorrect. Passage 2 contains nothing that resembles an "anecdote," or retelling of an interesting incident.

The passage below is excerpted from the introduction to a collection of essays published in 1994. My entry into Black womens history was serendipitous. In the preface to Black Women in America: An Historical Encyclopedia, I recount the story of exactly how Shirley Line Herd (who, in addition to teaching in the local school sys5 tem, was also president of the Indianapolis chapter of the National Council of Negro Women) successfully provoked me into changing my research and writing focus. Although I dedicate this volume to her and to her best friend, fellow

10

15

20

25

30

35

40

45

club woman and retired primary school teacher Virtea Downey, I still blush at the fact that I went to graduate school to become a historian in order to contribute to the Black Struggle for social justice and yet met her request to write a history of Black women in Indiana with condescension. I had never even thought about Black women as historical subjects with their own relations to a states history, and I thought her invitation and phone call extraordinarily intrusive. Only later did I concede how straightforward and reasonable had been her request to redress a historical omission. Black women were conspicuous by their absence. None of the social studies texts or state histories that Herd and Downey had used to teach their students made mention of the contributions of Black women. Since historians had left them out, Herd reasoned, only a real historian could put them in, and since I was the only tenured * Black woman historian in the state of Indiana at that time, the task was mine. Herd rejected my reservations and completely ignored my admonitions that she could not call up a historian and order a book the way you drive up to a fast-food restaurant and order a hamburger. In spite of my assertions of ignorance about the history of Black women in Indiana and my confession of having never studied the subject in any history course or examined any manuscript sources pertaining to their lives, Herd persevered. Black women, as historical subjects and agents, were as invisible to me as they had been to school textbook writers. Undaunted by my response, Herd demanded that I connect (thankfully without perfect symmetry) my biology and autobiography, my race and gender, my being a Black woman, to my skill as a historian, and write for her and for the local chapter members of the National Council a history of Black women in Indiana. I relented and wrote the book, When the Truth Is Told: Black Womens Culture and Community in Indiana, 1875-1950, as requested. In the process, I was both humbled and astounded by the array of rich primary source materials Herd, Downey, and the other club women had spent two years collecting. There were diaries, club notes, church souvenir booklets, photographs,

50

55

club minutes, birth, death, and marriage certificates, letters, and handwritten county and local histories. Collectively this material revealed a universe I never knew existed in spite of having lived with Black women all of my life . . . and being one myself. Or perhaps more accurately, I knew a universe of Black women existed. I simply had not envisioned its historical meaning.

* tenure: a permanent position, often granted to a teacher after a specified number of years of demonstrated competence

10 The primary purpose of the passage is to show how the author (A) discovered Black womens history when she was in graduate school (B) became a historian to help Black people in America achieve social justice (C) developed her research skills by undertaking a challenging project (D) became a more renowned scholar due to the influence of two interesting individuals (E) came to view Black women as a worthy subject for historical analysis ANSWERS AND EXPLANATIONS

Explanation for Correct Answer E : Choice (E) is correct. The passage mainly discusses the process by which the author comes to realize that Black women are a worthy subject of historical study. Explanation for Incorrect Answer A : Choice (A) is incorrect. The author explains that she did not discover her interest in Black women's history until she was already a tenured historian. It was only after her discussions with Herd and Downey that the author became interested in the subject.

Explanation for Incorrect Answer B : Choice (B) is incorrect. Although the author does mention that she "went to graduate school to become a historian in order to contribute to the Black Struggle for social justice," the passage was not written to support this claim. Explanation for Incorrect Answer C : Choice (C) is incorrect. The author may have honed her research skills while preparing for her book, but the primary purpose of the passage is to show the development of her beliefs as a historian. Explanation for Incorrect Answer D : Choice (D) is incorrect. Although the author does mention the impact of Herd and Downey on her project, there is no indication that her collaboration with them enhanced her status as a historian. 11 The first sentence indicates that the authors entry (line 1) was (A) troublesome but worthwhile (B) challenging but rewarding (C) fortunate and inevitable (D) unexpected but agreeable (E) startling and provocative ANSWERS AND EXPLANATIONS

Explanation for Correct Answer D : Choice (D) is correct. The author describes her introduction to Black women's history as "serendipitous," indicating that she regarded her "entry" into the field as both unexpected and fortunate. Explanation for Incorrect Answer A :

Choice (A) is incorrect. The author would almost certainly agree that her entry into Black women's history was "worthwhile," but the term "serendipitous" does not suggest that it was particularly "troublesome." Explanation for Incorrect Answer B : Choice (B) is incorrect. The first sentence alone does not support the claim that the author's initiation into Black women's history was "challenging." Explanation for Incorrect Answer C : Choice (C) is incorrect. "Serendipitous" means the opposite of "inevitable," or unavoidable. Explanation for Incorrect Answer E : Choice (E) is incorrect. The author may have been startled to find so much value in a topic that she had previously ignored, but "serendipitous" does not mean "provocative," or stimulating. 12 The author initially responded to Herds request with condescension (lines 13-14) because the author (A) knew that Herd had not been to graduate school (B) believed that historians should avoid controversial projects (C) had too many other projects requiring her attention (D) rejected Herds contention that such a history would address the Black struggle for social justice (E) viewed Herds request as irrelevant and presumptuous ANSWERS AND EXPLANATIONS

Explanation for Correct Answer E : Choice (E) is correct. The author clearly considered Herd's request "presumptuous" and "intrusive," but she also believed it to be irrelevant

because she "had never even thought about Black women as historical subjects." Explanation for Incorrect Answer A : Choice (A) is incorrect. The author does not mention Herd's educational credentials. Explanation for Incorrect Answer B : Choice (B) is incorrect. The author gives no indication that she thinks historians should avoid controversy. Explanation for Incorrect Answer C : Choice (C) is incorrect. The author does not specifically mention whether she had any other time-consuming commitments. Explanation for Incorrect Answer D : Choice (D) is incorrect. The author does not indicate that Herd explicitly discussed a "struggle for social justice." 13 The comparison in lines 27-30 (Herd . . . hamburger) primarily demonstrates the authors belief that historians (A) do not usually accept pay for their work (B) are frequently unassuming about their profession (C) do not generally undertake projects on request (D) spend a comparatively long time on their projects (E) do not generally interact with members of the public ANSWERS AND EXPLANATIONS

Explanation for Correct Answer C :

Choice (C) is correct. The author's admonishment indicates that she was not accustomed to undertaking academic projects upon request. Explanation for Incorrect Answer A : Choice (A) is incorrect. The author does not mention the issue of payment at any point in the passage. Explanation for Incorrect Answer B : Choice (B) is incorrect. The author does not imply that historians are modest about their work. On the contrary, these lines reflect her initial arrogance regarding Herd's proposed project. Explanation for Incorrect Answer D : Choice (D) is incorrect. While historians may spend many years working on one project, the hamburger comparison does not address this idea. Explanation for Incorrect Answer E : Choice (E) is incorrect. Although it may be rare for a historian to accept a project idea from a member of the public, there is no evidence to support the claim that historians do not "interact with members of the public" in general. 14 Lines 30-34 (In spite . . . persevered) suggest that the author believed that (A) her lack of scholarly training on this topic was a reason to be embarrassed (B) primary source materials on this subject would be difficult to find (C) historians should conduct research in the areas in which they have expertise (D) the lives of Black women in Indiana were historically interesting and complex (E) Herd wanted her to conduct research on a topic of general interest ANSWERS AND EXPLANATIONS

Explanation for Correct Answer C : Choice (C) is correct. The author tried to excuse herself from fulfilling Herd's request by professing her ignorance of Black women's history. Inherent in this action is the assumption that historians should work within their areas of expertise. Explanation for Incorrect Answer A : Choice (A) is incorrect. The author does not imply that her lack of scholarly training in this previously neglected subject is a source of embarrassment. Explanation for Incorrect Answer B : Choice (B) is incorrect. While the author had not studied any primary sources before accepting Herd's proposal, she does not imply that she had assumed they would be difficult to find. Explanation for Incorrect Answer D : Choice (D) is incorrect. The author clearly states that she had never before "thought about Black women as historical subjects." Explanation for Incorrect Answer E : Choice (E) is incorrect. Nothing in this sentence or in the passage as a whole suggests that Black women's history was considered to be a mainstream subject. 15 The last two sentences (Or perhaps . . . meaning) primarily indicate that the author (A) knew that Black women contributed to society, but she did not understand the significance of their contributions (B) believed that the diversity of Black womens experiences would make them difficult to write about

(C) assumed that because Black women are not frequently studied by historians, they would not be an acceptable topic for a book (D) believed that Black women wield political power only in contemporary times (E) was aware of the diversity of Black womens lives, but was not willing to write about them ANSWERS AND EXPLANATIONS

Explanation for Correct Answer A : Choice (A) is correct. The final two sentences of the passage clarify the author's original misconception of Black women's role in history. She writes that she had been aware of "a universe of Black women" without comprehending its historical significance. Explanation for Incorrect Answer B : Choice (B) is incorrect. The author does not indicate that the book was particularly challenging to write. Explanation for Incorrect Answer C : Choice (C) is incorrect. The passage asserts that Black women's history has been widely neglected by academics, but ultimately argues that it is a worthy book topic. Explanation for Incorrect Answer D : Choice (D) is incorrect. The author does not discuss the history of Black women's political power in these sentences or elsewhere in the passage. Explanation for Incorrect Answer E : Choice (E) is incorrect. The last two sentences are less about diversity than they are about Black women's historical significance.

This passage, from a short story published in 1978, describes a visit to a planetarium, a building in which images of stars, planets, and other astronomical phenomena are projected onto a domed ceiling. Inside, we sat on wonderfully comfortable seats that were tilted back so that you lay in a sort of a hammock, attention directed to the bowl of the ceiling, which soon Line turned dark blue, with a faint rim of light around the edge. 5 There was some splendid, commanding music. The adults all around were shushing the children, trying to make them stop crackling their potato chip bags. Then a mans voice, an eloquent professional voice, began to speak slowly, out of the walls. The voice reminded me a little of the way 10 radio announcers used to introduce a piece of classical music or describe the progress of the Royal Family to Westminster Abbey on one of their royal occasions. There was a faint echo-chamber effect. The dark ceiling was filled with stars. They came out not 15 all at once but one after another, the way stars really do come out at night, though more quickly. The Milky Way galaxy appeared, was moving closer; stars swam into brilliance and kept on going, disappearing beyond the edges of the sky-screen or behind my head. While the 20 flow of light continued, the voice presented the stunning facts. From a few light-years away, it announced, the Sun appears as a bright star, and the planets are not visible. From a few dozen light-years away, the Sun is not visible, either, to the naked eye. And that distancea few dozen 25 light-yearsis only about a thousandth part of the distance from the Sun to the center of our galaxy, one galaxy, which itself contains about two hundred billion stars. And is, in turn, one of millions, perhaps billions, of galaxies. Innumerable repetitions, innumerable variations. All this 30 rolled past my head, too, like balls of lightning. Now realism was abandoned, for familiar artifice. A model of the solar system was spinning away in its elegant style. A bright bug took off from the Earth, heading for Jupiter. I set my dodging and shrinking mind sternly 35 to recording facts. The mass of Jupiter two and a half times that of all the other planets put together. The Great Red Spot. The thirteen moons. Past Jupiter, a glance at

40

45

50

55

60

the eccentric orbit of Pluto, the icy rings of Saturn. Back to Earth and moving in to hot and dazzling Venus. Atmospheric pressure ninety times ours. Moonless Mercury rotating three times while circling the Sun twice; an odd arrangement, not as satisfying as what they used to tell us that it rotated once as it circled the Sun. No perpetual darkness after all. Why did they give out such confident information, only to announce later that it was quite wrong? Finally, the picture already familiar from magazines: the red soil of Mars, the blooming pink sky. When the show was over I sat in my seat while children clambered over me, making no comments on anything they had just seen or heard. They were pestering their keepers for eatables and further entertainments. An effort had been made to get their attention, to take it away from canned drinks and potato chips and fix it on various knowns and unknowns and horrible immensities, and it seemed to have failed. A good thing, too, I thought. Children have a natural immunity, most of them, and it shouldnt be tampered with. As for the adults who would deplore it, the ones who promoted this show, werent they immune themselves to the extent that they could put in the echo-chamber effects, the music, the solemnity, simulating the awe that they supposed they ought to feel? Awewhat was that supposed to be? A fit of the shivers when you looked out the window? Once you knew what it was, you wouldnt be courting it.

16 Which best describes the overall structure of the passage? (A) Narrative description followed by commentary (B) Reminiscence followed by present-day application (C) An account of a problem followed by a suggested solution (D) A generalization followed by specific examples (E) A discussion of opposing viewpoints followed by an attempt to reconcile them ANSWERS AND EXPLANATIONS

Explanation for Correct Answer A : Choice (A) is correct. The bulk of the passage recreates the narrator's visit to a planetarium, but the text switches from "description" to "commentary" in the final paragraph. Explanation for Incorrect Answer B : Choice (B) is incorrect. The passage seems to be a "reminiscence," or a recollection, of past events, but there is no indication that the narrator jumps from the past to the present in either verb tense or content. Explanation for Incorrect Answer C : Choice (C) is incorrect. The narrator suggests that the pursuit of "awe" is fundamentally problematic but does not offer a "solution" of any kind. Explanation for Incorrect Answer D : Choice (D) is incorrect. The narrator uses a myriad of details, not generalizations, to depict the sensory experience created inside of the planetarium. Explanation for Incorrect Answer E : Choice (E) is incorrect. The only viewpoint presented in the passage is that of the narrator. 17 Lines 5-7 (The adults . . . bags) primarily illustrate the childrens feelings of (A) helplessness (B) restlessness (C) awe (D) anticipation (E) irritation ANSWERS AND EXPLANATIONS

Explanation for Correct Answer B : Choice (B) is correct. "Restless" means antsy or excitable. The image of "adults all around . . . shushing the children" suggests that the children are too restless to remain quiet in the darkened room. Explanation for Incorrect Answer A : Choice (A) is incorrect. The adults may feel helpless in their efforts to quiet the noisy children, but nothing in these lines indicates that the children themselves are helpless. Explanation for Incorrect Answer C : Choice (C) is incorrect. The children's agitation contrasts with the feelings of "awe" that the planetarium show is presumably meant to inspire. Explanation for Incorrect Answer D : Choice (D) is incorrect. While some of the children in the planetarium audience may look forward to the show, their general noisiness indicates inattention more than "anticipation." Explanation for Incorrect Answer E : Choice (E) is incorrect. Some of the children may act out of mild boredom, but the narrator does not suggest that they are largely irritated, or annoyed. 18 In line 11, progress most nearly means (A) (B) (C) (D) (E) evolution improvement prosperity promotion advance

ANSWERS AND EXPLANATIONS

Explanation for Correct Answer E : Choice (E) is correct. The word "progress" in this context means a ceremonial journey. Line 11 refers to members of British royalty making such a trip to a famous London cathedral. Explanation for Incorrect Answer A : Choice (A) is incorrect. The term "progress" is sometimes used to refer to "evolution," or gradual development, but it is illogical to describe the "evolution" of a group of people to a place. Explanation for Incorrect Answer B : Choice (B) is incorrect. "Progress" sometimes indicates a positive change, but it makes little sense to describe the Royal Family's procession to Westminster Abbey as an "improvement." Explanation for Incorrect Answer C : Choice (C) is incorrect. While it may be said that one who prospers is progressing financially or personally, this use of the term "progress" does not fit in the context of line 11. Explanation for Incorrect Answer D : Choice (D) is incorrect. The term "promotion" typically refers to a professional progression, which has nothing to do with the Royal Family's ceremonial journey. 19 The first paragraph of the passage establishes a mood of (A) (B) jaded dismissal nervous apprehension

(C) dramatic anticipation (D) initial concern (E) mundane routine ANSWERS AND EXPLANATIONS

Explanation for Correct Answer C : Choice (C) is correct. The first paragraph attempts to recreate the tension that exists in a darkened planetarium as audience members settle into their chairs and wait for the show to begin. The author's evocative description of the domed screen coming to life as "commanding music" swells under an "eloquent," disembodied voice contributes to the feeling of "dramatic anticipation." Explanation for Incorrect Answer A : Choice (A) is incorrect. At no point does the narrator appear to be "jaded," or tired, of the planetarium experience. Explanation for Incorrect Answer B : Choice (B) is incorrect. The first paragraph is peppered with adjectives such as "comfortable," "splendid," and "eloquent." These terms create a mood that is neither nervous nor apprehensive. Explanation for Incorrect Answer D : Choice (D) is incorrect. The passage does not suggest that the narrator has any concerns about the upcoming presentation. Explanation for Incorrect Answer E : Choice (E) is incorrect. It is unlikely that an author would describe a mundane, routine event or feeling in such colorful detail. 20

The words dodging and shrinking (line 34) primarily suggest that the author was (A) somewhat bothered by the children in the audience (B) initially overwhelmed by the information being presented (C) unable to admit to some troubling feelings about astronomy (D) refusing to acknowledge the implications of space travel (E) unwilling to believe the studies being discussed ANSWERS AND EXPLANATIONS

Explanation for Correct Answer B : Choice (B) is correct. "Stunning facts" illustrating the immensity of the universe "rolled past" the narrator's head in the second paragraph, creating a dizzying web of information. The narrator's mind seems to dodge and shrink away from the deluge of figures and concepts. Only when the facts become less abstract and more familiar is the narrator capable of mechanically recording information. Explanation for Incorrect Answer A : Choice (A) is incorrect. The reference to the narrator's "dodging and shrinking mind" appears in line 34. It is unlikely that the children who are briefly mentioned in line 6 are responsible for the narrator's mental state almost 30 lines later. Explanation for Incorrect Answer C : Choice (C) is incorrect. The narrator readily admits that the astronomical facts are "troubling" in their complexity. Explanation for Incorrect Answer D : Choice (D) is incorrect. No significant mention of "space travel" appears at any point in the passage. Explanation for Incorrect Answer E :

Choice (E) is incorrect. Although the narrator acknowledges in lines 4445 that "confident" scientific information is often revised, nothing in the passage suggests that the narrator's "dodging and shrinking mind" is related to any doubts regarding the show's accuracy. 21 In lines 40-43 (Moonless . . . Sun), the narrators comment about the arrangement demonstrates a preference for (A) irony (B) inventiveness (C) symmetry (D) ornamentation (E) ambiguity ANSWERS AND EXPLANATIONS

Explanation for Correct Answer C : Choice (C) is correct. The narrator is disappointed to learn from the planetarium show that Mercury's orbit is more complicated than once thought. Apparently the moonless planet rotates three times per every two trips around the sun, "an odd arrangement" compared to the outdated theory that Mercury rotates just once per solar orbit. The narrator's preference for the simpler "arrangement" implies a preference for "symmetry," or balance. Explanation for Incorrect Answer A : Choice (A) is incorrect. Lines 4043 do not demonstrate the narrator's sense of "irony." Explanation for Incorrect Answer B : Choice (B) is incorrect. The narrator's reaction to Mercury's "odd arrangement" may be unusual, but it does not particularly demonstrate "inventiveness," or creativity. Explanation for Incorrect Answer D :

Choice (D) is incorrect. The narrator includes numerous details in the passage but presents them simply. It is somewhat inaccurate, then, to claim that the narrator's style as it appears in lines 4043 is ornamental, or embellished. Explanation for Incorrect Answer E : Choice (E) is incorrect. The narrator's feelings regarding Mercury's complicated orbit suggest a discomfort with "ambiguity," or confusion, not a preference for it. 22 In line 53, fix most nearly means (A) focus (B) prepare (C) repair (D) decide (E) influence ANSWERS AND EXPLANATIONS

Explanation for Correct Answer A : Choice (A) is correct. "Fix" in this context means to direct one's attention. Lines 5155 specifically refer to the adults' effort to catch the children's attention and direct it to the planetarium show. Explanation for Incorrect Answer B : Choice (B) is incorrect. "Fix" sometimes means to "prepare," but this definition is illogical within the context of line 53. Explanation for Incorrect Answer C :

Choice (C) is incorrect. A common definition of "fix" is to "repair," but it is illogical to claim that adults tried to "repair" children's attention to something. Explanation for Incorrect Answer D : Choice (D) is incorrect. The term "fix" is occasionally used to reflect a decision, but a child's attention cannot be decided on something else. Explanation for Incorrect Answer E : Choice (E) is incorrect. An event or outcome that has been improperly influenced is sometimes described as "fixed," but this sense of the word does not fit the context of line 53. 23 The phrase horrible immensities (line 54) primarily indicates (A) exaggerated information (B) unforeseen events (C) historical monstrosities (D) controversial debates (E) incomprehensible realities ANSWERS AND EXPLANATIONS

Explanation for Correct Answer E : Choice (E) is correct. The astronomical "realities" portrayed in the planetarium show suggest a universe so immense that its sheer size is "incomprehensible" and horribly intimidating to the narrator. Explanation for Incorrect Answer A : Choice (A) is incorrect. Facts that indicate the vastness of the universe may seem "exaggerated," but there is nothing to suggest that the information presented at the planetarium is overstated.

Explanation for Incorrect Answer B : Choice (B) is incorrect. The presentation does not focus on "unforeseen," or unexpected, events. Explanation for Incorrect Answer C : Choice (C) is incorrect. The presentation does not address a single monstrosity, historical or otherwise. Explanation for Incorrect Answer D : Choice (D) is incorrect. Nowhere in the passage does the narrator directly refer to "controversial debates." 24 The author suggests that the echo-chamber effects, the music, the solemnity (lines 59-60) are evidence that (A) most adults have feelings of great appreciation for the universe (B) most adults would rather not attend planetarium shows (C) contemporary scientists have an inflated view of the importance of their work (D) the shows promoters do not fully appreciate the true nature of the universe (E) the shows promoters understand that children are entranced by special effects ANSWERS AND EXPLANATIONS

Explanation for Correct Answer D : Choice (D) is correct. The narrator criticizes the show's promoters by suggesting that they used special effects such as "echo-chamber effects" to create a mere simulation of "the awe that they.. . ought to feel" at the immensity of the universe. Explanation for Incorrect Answer A :

Choice (A) is incorrect. The narrator does not imply that the majority of adults appreciate the universe's "horrible immensities," but instead suggests that the adults responsible for infusing the show with special effects are "immune themselves" to the awesome realities of the universe. Explanation for Incorrect Answer B : Choice (B) is incorrect. The narrator does not speculate about the popularity of planetarium shows among adults. Explanation for Incorrect Answer C : Choice (C) is incorrect. Contemporary scientists likely contributed to the planetarium presentation, but the narrator does not discuss how these scientists feel about their work. Explanation for Incorrect Answer E : Choice (E) is incorrect. The show's creators may have included special effects to appeal to children, but the narrator does not mention such a theory. The only implication is that such effects illustrate the promoters' immunity to the awesome nature of the universe.

Section 8: Math Section 9: Critical Reading


1 The new vaccine is ------- preventing certain forms of pneumonia and should, therefore, be more widely ------- in order to prevent outbreaks of the disease. (A) required for . . constrained (B) unsuccessful in . . distributed (C) instrumental in . . reconstituted (D) effective in . . administered (E) unverified for . . disseminated ANSWERS AND EXPLANATIONS

Explanation for Correct Answer D : Choice (D) is correct. Something that is "effective" produces a desired effect. "Administered" means dispensed. If one were to insert these terms into the text, the sentence would read "The new vaccine is effective in preventing certain forms of pneumonia and should, therefore, be more widely administered in order to prevent outbreaks of the disease." The word "therefore" indicates that the second part of the sentence should logically occur, based on the claim made in the first part. It makes sense to argue that a successful vaccine should be widely administered. Explanation for Incorrect Answer A : Choice (A) is incorrect. "Required" means needed or essential. "Constrained" means to keep within close bounds. If one were to insert these terms into the text, the sentence would read "The new vaccine is required for preventing certain forms of pneumonia and should, therefore, be more widely constrained in order to prevent outbreaks of the disease." It makes no sense to say that a vaccine that is required for preventing illness should be "constrained," or restricted. Explanation for Incorrect Answer B : Choice (B) is incorrect. "Unsuccessful" means having an unfavorable outcome. "Distributed" means delivered or handed out. If one were to insert these terms into the text, the sentence would read "The new vaccine is unsuccessful in preventing certain forms of pneumonia and should, therefore, be more widely distributed in order to prevent outbreaks of the disease." It is illogical to argue that an unsuccessful vaccine should be distributed or passed out more widely. Explanation for Incorrect Answer C : Choice (C) is incorrect. "Instrumental" means serving as a means or agency. "Reconstituted" means provided with a new structure. If one were to insert these terms into the text, the sentence would read "The new vaccine is instrumental in preventing certain forms of pneumonia and should, therefore,

be more widely reconstituted in order to prevent outbreaks of the disease." An instrumental or helpful vaccine does not need to be reconstituted, or remade, because it is already effective. Explanation for Incorrect Answer E : Choice (E) is incorrect. "Unverified" means lacking proof or substantiation. "Disseminated" means dispersed throughout. If one were to insert these terms into the text, the sentence would read "The new vaccine is unverified for preventing certain forms of pneumonia and should, therefore, be more widely disseminated in order to prevent outbreaks of the disease." It makes no sense to argue that an unverified or unproven vaccine should be disseminated or dispensed more widely. 2 In an effort to ------- the ------- theater, the troupe members contributed thousands of dollars to keep the playhouse operating. (A) qualify . . obsolete (B) salvage . . floundering (C) exacerbate . . defunct (D) revitalize . . prosperous (E) commandeer . . lucrative ANSWERS AND EXPLANATIONS

Explanation for Correct Answer B : Choice (B) is correct. "Salvage" means to save from ruin, and "floundering" means struggling. If one were to insert these terms into the text, the sentence would read "In an effort to salvage the floundering theater, the troupe members contributed thousands of dollars to keep the playhouse operating." The fact that the theater required thousands of dollors to maintain operations implies that it was floundering and needed to be salvaged. Explanation for Incorrect Answer A :

Choice (A) is incorrect. "Qualify" means to describe or to modify. "Obsolete" means no longer in use. If one were to insert these terms into the text, the sentence would read "In an effort to qualify the obsolete theater, the troupe members contributed thousands of dollars to keep the playhouse operating." The second half of the sentence indicates that the theater is failing, but not yet obsolete. Furthermore, the verb "qualify" does not make sense in this context. Explanation for Incorrect Answer C : Choice (C) is incorrect. "Exacerbate" means to make worse, and something that is "defunct" has ceased to exist. If one were to insert these terms into the text, the sentence would read "In an effort to exacerbate the defunct theater, the troupe members contributed thousands of dollars to keep the playhouse operating." It is illogical to assert that actors would donate money to a struggling theater in an effort to make its problems worse. Explanation for Incorrect Answer D : Choice (D) is incorrect. "Revitalize" means to give new life to, and "prosperous" means marked by economic well-being. If one were to insert these terms into the text, the sentence would read "In an effort to revitalize the prosperous theater, the troupe members contributed thousands of dollars to keep the playhouse operating." A "prosperous," or wealthy, theater would not need to be revitalized through financial donations. Explanation for Incorrect Answer E : Choice (E) is incorrect. "Commandeer" means to take by force, and "lucrative" means profitable. If one were to insert these terms into the text, the sentence would read "In an effort to commandeer the lucrative theater, the troupe members contributed thousands of dollars to keep the playhouse operating." It is unlikely that actors would attempt to forcibly take over a playhouse or that a "lucrative" theater would be in need of financial donations. 3

In her writings about language, the poet Gloria Anzalda celebrates the ------- of English and Spanish dialects spoken by Mexican Americans, arguing that such ------- lends an empowering flexibility to expression. (A) multiplicity . . variety (B) proliferation . . moderation (C) ambivalence . . focus (D) dearth . . depletion (E) abridgment . . imitation ANSWERS AND EXPLANATIONS

Explanation for Correct Answer A : Choice (A) is correct. "Multiplicity" means a great number. "Variety" means something varying from others of the same general kind. If one were to insert these terms into text, the sentence would read "In her writings about language, the poet Gloria Anzalda celebrates the multiplicity of English and Spanish dialects spoken by Mexican Americans, arguing that such variety lends an empowering flexibility to expression." The word "such" suggests that the two blanked words are synonyms, both describing a celebrated feature of Mexican American speech. "Multiplicity" and "variety" describe Mexican American language in the same way. Multiple and various forms of speech would also give Mexican Americans the "empowering flexibility of expression" Anzalda refers to. Explanation for Incorrect Answer B : Choice (B) is incorrect. Proliferation means to increase at a rapid rate. Moderation means being within reasonable limits. If one were to insert these terms into the text, the sentence would read In her writings about language, the poet Gloria Anzalda celebrates the proliferation of English and Spanish dialects spoken by Mexican Americans, arguing that such moderation lends an empowering flexibility to expression. A proliferation, or rapid increase, of dialects would not be described as a moderation. It is also unlikely that moderation could give someone flexibility of expression.

Explanation for Incorrect Answer C : Choice (C) is incorrect. "Ambivalence" means uncertainty or indecisiveness. "Focus" means a center of interest or activity. If one were to insert these terms into text, the sentence would read "In her writings about language, the poet Gloria Anzalda celebrates the ambivalence of English and Spanish dialects spoken by Mexican Americans, arguing that such focus lends an empowering flexibility to expression." "Ambivalence," or conflicting feelings, would tend to take away from a person's focus. Furthermore, the two words are not synonyms. Explanation for Incorrect Answer D : Choice (D) is incorrect. "Dearth" means a lack of or scarce supply. "Depletion" means the act of depleting. If one were to insert these terms into text, the sentence would read "In her writings about language, the poet Gloria Anzalda celebrates the dearth of English and Spanish dialects spoken by Mexican Americans, arguing that such depletion lends an empowering flexibility to expression." While "dearth" and "depletion" both refer to a lack of something, it is illogical to state that a lack of ways of speaking would lend "flexibility to expression." Explanation for Incorrect Answer E : Choice (E) is incorrect. "Abridgment" means the act of reducing or shortening. "Imitation" means something derived or copied from an original. If one were to insert these terms into text, the sentence would read "In her writings about language, the poet Gloria Anzalda celebrates the abridgement of English and Spanish dialects spoken by Mexican Americans, arguing that such imitation lends an empowering flexibility to expression."An "abridgment" is a reduction or shortening of something, a concept that has little to do with either "imitation" or "flexibility of expression." 4 The mountain road was distinctly -------: it twisted back and forth along the contours of the hillside. (A) panoramic

(B) precipitous (C) serpentine (D) circumscribed (E) retrograde ANSWERS AND EXPLANATIONS

Explanation for Correct Answer C : Choice (C) is correct. "Serpentine" means winding. If one were to insert this term into the text, the sentence would read "The mountain road was distinctly serpentine: it twisted back and forth along the contours of the hillside." The colon indicates that the second part of the sentence will illustrate the meaning of the missing term. "Serpentine" fits the sentence because a winding road does indeed twist "back and forth." Explanation for Incorrect Answer A : Choice (A) is incorrect. "Panoramic" refers to a complete view of a surrounding area. If one were to insert this term into the text, the sentence would read "The mountain road was distinctly panoramic: it twisted back and forth along the contours of the hillside." A winding road will not necessarily offer "panoramic" views. Explanation for Incorrect Answer B : Choice (B) is incorrect. "Precipitous" means extremely steep. If one were to insert this term into the text, the sentence would read "The mountain road was distinctly precipitous: it twisted back and forth along the contours of the hillside." There is no indication that the winding road in the sentence is particularly steep. Explanation for Incorrect Answer D : Choice (D) is incorrect. "Circumscribed" refers to something that is constricted or surrounded by a boundary. If one were to insert this term into the text, the sentence would read "The mountain road was distinctly circumscribed: it twisted back and forth along the contours of the hillside."

The statement after the colon does not indicate that the road is "circumscribed" in any way. Explanation for Incorrect Answer E : Choice (E) is incorrect. "Retrograde" means moving backward. If one were to insert this term into the text, the sentence would read "The mountain road was distinctly retrograde: it twisted back and forth along the contours of the hillside." It does not make sense to describe the road as moving in a backward direction. 5 At the family reunion Hiroko found her cousin charming and gentle, the ------- of his formerly rude and overbearing self. (A) remnant (B) antithesis (C) consequence (D) extremity (E) mainstay ANSWERS AND EXPLANATIONS

Explanation for Correct Answer B : Choice (B) is correct. "Antithesis" means exact opposite. If one were to insert this term into the text, the sentence would read "At the family reunion Hiroko found her cousin charming and gentle, the antithesis of his formerly rude and overbearing self." The sentence indicates that Hiroko's cousin has changed dramatically. He was once "rude and overbearing," and is now "charming and gentle." "Antithesis" connotes this dramatic change perfectly. Explanation for Incorrect Answer A : Choice (A) is incorrect. "Remnant" means a remainder. If one were to insert this term into the text, the sentence would read "At the family reunion Hiroko found her cousin charming and gentle, the remainder of his formerly rude and overbearing self." It makes no sense to say that "charming and

gentle" qualitites would be the "remnant," or remainder, of a "formerly rude and overbearing self." Explanation for Incorrect Answer C : Choice (C) is incorrect. "Consequence" means the relation of a result to its cause. If one were to insert this term into the text, the sentence would read "At the family reunion Hiroko found her cousin "charming and gentle," the consequence of his formerly rude and overbearing self." It is illogical to claim that charming and gentle behavior could be a consequence of "rude and overbearing" behavior. This sentence suggests that Hiroko's cousin's behavior is different from what it once was, not that it is the result of what it once was. Explanation for Incorrect Answer D : Choice (D) is incorrect. "Extremity" means the farthest point. If one were to insert this term into the text, the sentence would read "At the family reunion Hiroko found her cousin charming and gentle, the extremity of his formerly rude and overbearing self." Hiroko's cousin's new behavior is not an extreme expression of her cousin's former self, it is completely different. Explanation for Incorrect Answer E : Choice (E) is incorrect. "Mainstay" means a chief support or foundational influence or characteristic. If one were to insert this term into the text, the sentence would read "At the family reunion Hiroko found her cousin charming and gentle, the mainstay of his formerly rude and overbearing self." Hiroko's cousin was exhibiting characteristics that seemed new, not foundational, to how his cousin previously knew him. 6 His conduct at the state dinner was a cavalcade of blunders, one ------following another until the evening ended. (A) (B) (C) (D) query gibe gaffe tryst

(E) tribute ANSWERS AND EXPLANATIONS

Explanation for Correct Answer C : Choice (C) is correct. "Gaffe" means a blunder. If one were to insert this term into the text, the sentence would read "His conduct at the state dinner was a cavalcade of blunders, one gaffe following another until the evening ended." The comma indicates that the missing term is synonymous with blunder. Both "blunder" and "gaffe" can be used to refer to a social error. Explanation for Incorrect Answer A : Choice (A) is incorrect. A "query" is a question or inquiry. If one were to insert this term into the text, the sentence would read "His conduct at the state dinner was a cavalcade of blunders, one query following another until the evening ended." It is inaccurate to equate a blunder, or mistake, with a query. Explanation for Incorrect Answer B : Choice (B) is incorrect. "Gibe" means a derisive remark. If one were to insert this term into the text, the sentence would read "His conduct at the state dinner was a cavalcade of blunders, one gibe following another until the evening ended." The use of gibes at a formal dinner could certainly be considered to be a social faux pas, but a blunder does not always appear in the form of a taunt. Explanation for Incorrect Answer D : Choice (D) is incorrect. "Tryst" typically refers to a romantic meeting. If one were to insert this term into the text, the sentence would read "His conduct at the state dinner was a cavalcade of blunders, one tryst following another until the evening ended." The term "tryst" is not a synonym of "blunder." Explanation for Incorrect Answer E :

Choice (E) is incorrect. A "tribute" is an acknowledgment of gratitude, respect, or admiration. If one were to insert this term into the text, the sentence would read "His conduct at the state dinner was a cavalcade of blunders, one tribute following another until the evening ended." It's not likely that a tribute at a state dinner would be described as a social error.

Both passages discuss the issue of the intelligence of dogs. Passage 1 was adapted from a 2001 book on animal intelligence. Passage 2 was written in 2001 by a dog trainer and writer. Passage 1 It was no accident that nineteenth-century naturalist Charles Darwin strove to connect the mentality and emotionality of people with that of dogs, rather than, say, Line doves or horses. Neither his theory of evolution nor any 5 general understanding of biology demanded that he preferentially underline our similarity to dogs over other species. But politically and emotionally, the choice was inevitable for an English gentleman who had set himself the task of making the idea of evolutionary continuity 10 palatable. Darwin wrote that dogs possess something very similar to a conscience. They certainly possess some power of self-command. . . . Dogs have long been accepted as the very type of fidelity and obedience. Darwin was not alone in his beliefs that dogs possess 15 human virtues. The characteristics of loyalty and obedience, coupled with an expressive face and body, can account for why dogs are such popular and valued pets in many cultures. Depending on the breed and the individual, dogs can be noble, charming, affectionate, and reliable. But 20 while all dog owners should rightly appreciate these and other endearing traits in their pets, nothing says that the cleverness of a highly intelligent primate such as a chimpanzee is part of the package. Scientists generally believe the reasoning abilities of chimps to be considerably greater 25 than that of dogs. But many people nonetheless believe that dogs are smarter than chimps precisely because of our

30

35

familiarity and emotional ties with the dogs that we love. We apply the same secret rules to our fellow humans: the old in-group, out-group story. People in your in-group are those who are similar to you, either because they belong to the same organizations as you, or enjoy the same activities, or, and this is the kicker, because they are simply around more often. Dogs, because of their proximity to their owners, are definitely in. The intensity of our relationship with dogs causes us, quite naturally, to imbue them with high-level mental abilities, whether they have earned those extra intelligence points or not. We like them, so we think well of them.

Passage 2 Every dog trainer that I know had the same childhood, a childhood filled with the brilliant, heroic dogs of literature. We read about dogs who regularly traveled thousands of miles to be reunited with owners who somehow misplaced them, repeatedly saved people from certain death, and continually exhibited a better grasp of strategic problemsolving than the average Ph.D. In the preface to one of his many dog stories, S. P. Meek a bit shamefacedly remarked that in writing of dogs I endeavored to hold these heroes down to the level of canine intelligence, and to make them, above all, believable. If at times I seem to have made them show supercanine intelligence, it is because my enthusiasm has run away with me. We forgave him, of course. It was something of a shock, therefore, to discover how the learning theory experts believed dogs think and learn. I was told that dogs, unlike chimpanzees, have no real reasoning ability. Dogs dont think: rather, they learn to avoid the unpleasant (negative reinforcement), seek the pleasant (positive reinforcement), or some combination of the two. To contend otherwise was to be guilty of the sin of anthropomorphizing, the attribution to an animal of motivations and consciousness that only a human being could possess. Yet as a dog trainer, I find myself siding more with the

40

45

50

55

60

65

70

75

80

Meeks than I do with the learning theorists: nobody could believe dispassionately in the totality of positive and negative reinforcement after seeing the pure intelligence shining in the face of a border collie intent upon helping a shepherd herd sheep. Dogs do think and reason. Granted, a dog might not be able to run a maze as quickly as a chimp. But a dog outshines any other animal that I know in the ability to work willingly with a human being, to communicate with a puzzling creature who often makes incomprehensible demands. Researchers have increasingly come to view intelligence as a complex collection of mental abilities that cannot be fully captured in any simple way. Dogs are geniuses at being useful, and it is this usefulness that we admire when we praise their intelligence. As Jonica Newby, a specialist in animal-human interaction, writes, In some ways intelligence is a matter of matching behavior to environment. To compare intelligence in creatures that have evolved differently is a bit like deciding which has hit upon the best mode of travel: the dolphin or the horse. And it is dogs, not chimps, who possess the most helpful mode of travel for human beings.

7 Unlike the author of Passage 1, the author of Passage 2 develops an argument by relying on (A) personal experience (B) scientific observation (C) historical contextualization (D) statistical evidence (E) direct quotation ANSWERS AND EXPLANATIONS

Explanation for Correct Answer A : Choice (A) is correct. Passage 1 examines the issue of dog intelligence from a primarily objective perspective, while Passage 2 is infused with the author's first-hand experiences.

Explanation for Incorrect Answer B : Choice (B) is incorrect. Passage 1 relies heavily on "scientific" evidence. Passage 2 does not. Explanation for Incorrect Answer C : Choice (C) is incorrect. Unlike the author of Passage 1, the author of Passage 2 provides little "historical" background information. Explanation for Incorrect Answer D : Choice (D) is incorrect. The author of Passage 2 does not provide "statistical evidence." Explanation for Incorrect Answer E : Choice (E) is incorrect. Both authors use direct quotations to develop their arguments. 8 The phrase It was no accident (line 1) implies that the author of Passage 1 believes that Darwin (A) knew that the resemblance between dogs and humans could not be accounted for by his theory (B) exploited the sympathies of his audience to gain support for his theory (C) considered intelligence to be largely a matter of luck (D) believed that the way previous authors had written about dogs was inaccurate (E) wished to convince the public to celebrate the virtues of dogs ANSWERS AND EXPLANATIONS

Explanation for Correct Answer B :

Choice (B) is correct. The author suggests that Darwin's example of animal intelligence was intentional. His readers' tendency to hold dogs in high regard made them more likely to accept his theory of evolutionary continuity. Explanation for Incorrect Answer A : Choice (A) is incorrect. Darwin's theory does in fact account for the similarities between dogs and humans. Explanation for Incorrect Answer C : Choice (C) is incorrect. The phrase "[i]t was no accident" has no connection to the question of whether or not intelligence is a matter of luck. Explanation for Incorrect Answer D : Choice (D) is incorrect. The author does not mention Darwin's opinions regarding previous depictions of dogs. Explanation for Incorrect Answer E : Choice (E) is incorrect. Darwin's focus is on supporting his argument, not on celebrating animals. 9 In line 13, type most nearly means (A) category (B) model (C) designation (D) role (E) figure ANSWERS AND EXPLANATIONS

Explanation for Correct Answer B :

Choice (B) is correct. Darwin describes dogs as "the very type of fidelity and obedience," implying that they are a "model," or ideal example, of these qualities. Explanation for Incorrect Answer A : Choice (A) is incorrect. Dogs may embody "fidelity and obedience," but it is illogical to claim that they are a "category," or division, of specific behavorial traits. Explanation for Incorrect Answer C : Choice (C) is incorrect. While Darwin implies that dogs exemplify certain traits, it would be illogical to claim that they are a "designation," or name, of "fidelity and obedience." Explanation for Incorrect Answer D : Choice (D) is incorrect. Dogs may have played a "role" in Darwin's theories regarding animal intelligence, but it would make little sense to say that they are the "very role" of certain behavioral traits. Explanation for Incorrect Answer E : Choice (E) is incorrect. The term "figure" is not typically used in this context. 10 The italics in line 25 most directly emphasize (A) a misguided idea that is becoming obsolete (B) a negative view that scientists adopt toward lay people (C) a common criticism of dog owners (D) the controversial nature of an alternative theory (E) the intensity of a conviction based on sentiment ANSWERS AND EXPLANATIONS

Explanation for Correct Answer E : Choice (E) is correct. Italics draw attention to the word "believe," emphasizing people's willingness to credit the dogs they love with a depth of intelligence that they may not possess. Explanation for Incorrect Answer A : Choice (A) is incorrect. Passage 1 suggests that the belief expressed in line 25 is not scientifically valid, or misguided. The italics in this sentence do not, however, indicate that public opinion on canine intelligence is changing or going away. Explanation for Incorrect Answer B : Choice (B) is incorrect. The author does not discuss the scientists' attitudes regarding the general public at any point in the passage. Explanation for Incorrect Answer C : Choice (C) is incorrect. The author alludes to dog owners' opinions but does not criticize them by placing the word "believe" in italics. Explanation for Incorrect Answer D : Choice (D) is incorrect. The use of italics does not indicate whether or not dog owners' beliefs are controversial. 11 In line 29, the author of Passage 1 uses the word old to suggest that the story is (A) familiar (B) historic (C) fictitious (D) tiresome (E) outdated ANSWERS AND EXPLANATIONS

Explanation for Correct Answer A : Choice (A) is correct. The author uses the word "old" in the phrase "the old in-group, out-group story" to suggest that this explanation is a familiar one. Explanation for Incorrect Answer B : Choice (B) is incorrect. The word "old" can certainly mean "historic" in certain contexts, but the author of passage 1 does not indicate that this "story" has historical significance of any kind. Explanation for Incorrect Answer C : Choice (C) is incorrect. This story is hardly "fictitious," or false; on the contrary, the author suggests that it accurately describes the bond between humans and dogs. Explanation for Incorrect Answer D : Choice (D) is incorrect. The author does not suggest in any way that the "story" referred to is "tiresome." Explanation for Incorrect Answer E : Choice (E) is incorrect. Something that is "outdated" is no longer current, but the author suggests that "the old in-group, out-group story" is just as useful in the present as it was in the past. 12 Darwin (lines 1-13, Passage 1) and Meek (lines 45-51, Passage 2) serve as examples of (A) (B) (C) (D) writers who had the courage to voice unpopular viewpoints researchers who conducted important studies on animal behavior people who maliciously publicized misleading information about dogs individuals whose writings reflect an idealized view of dog behavior

(E) scientists who were authorities on the issue of animal intelligence ANSWERS AND EXPLANATIONS

Explanation for Correct Answer D : Choice (D) is correct. Darwin's descriptions of dogs' supreme "fidelity and obedience" and Meek's tendency to bestow a "'supercanine intelligence'" on the dogs in his stories support the claim that these writers idealize dog behavior. Explanation for Incorrect Answer A : Choice (A) is incorrect. Darwin's efforts to make "the idea of evolutionary continuity palatable" indicate that some of his theories may have been "unpopular." There is no indication, however, that Meek's viewpoints were not well received. Explanation for Incorrect Answer B : Choice (B) is incorrect. Although Darwin was a researcher, Passage 2 only indicates that Meek was a writer of "many dog stories." Explanation for Incorrect Answer C : Choice (C) is incorrect. There is no indication that either Darwin or Meek deliberately misled the public about canine intelligence, and nothing in these passages suggests that the men were malicious. Explanation for Incorrect Answer E : Choice (E) is incorrect. Darwin might qualify as an authority on animal intelligence, but Meek seems to be a fiction writer. 13 In line 53, the author of Passage 2 uses quotation marks to (A) express anger about a prevailing belief

(B) demonstrate respect for a certain group of scientists (C) indicate uncertainty about the precise usage of a word (D) cite a term used in an unusual context (E) cast doubt on the aptness of a description ANSWERS AND EXPLANATIONS

Explanation for Correct Answer E : Choice (E) is correct. Writers sometimes place quotation marks around a word or phrase to cast doubt on its accuracy. The learning theory experts' "shocking" assessment of canine intelligence prompts the author of passage 2 to diminish the credibility of the learning theorists by placing the word "experts" in quotation marks. Explanation for Incorrect Answer A : Choice (A) is incorrect. It can be argued that the author's use of quotation marks expresses disbelief or even annoyance, but there is no conveyance of an extreme emotion such as anger. Explanation for Incorrect Answer B : Choice (B) is incorrect. The use of quotation marks around the word "experts" is a sign of skepticism, not "respect." Explanation for Incorrect Answer C : Choice (C) is incorrect. The author may view the experts' theories with uncertainty, but there is no reason to question the straightforward use of the word "experts" in this context. Explanation for Incorrect Answer D : Choice (D) is incorrect. It is not surprising to encounter the term "experts" within a discussion of a particular scientific theory. 14

The experts (line 53) would most likely argue that which of the following is guilty of the sin mentioned in line 58 ? (A) A veterinarian who is unwilling to treat a sick animal (B) A cat owner who believes his cat misses its siblings (C) A dog owner who is unwilling to punish her dog for misbehaving (D) A zoologist who places the interests of people before those of animals (E) A horse trainer who fails to recognize that his horse is hungry ANSWERS AND EXPLANATIONS

Explanation for Correct Answer B : Choice (B) is correct. This is a prime example of "anthropomorphizing," or attributing human feelings to animals. Explanation for Incorrect Answer A : Choice (A) is incorrect. The veterinarian's motivation for refusing to treat the animal would need to be specified to suggest anthropomorphization. Explanation for Incorrect Answer C : Choice (C) is incorrect. There is no indication that the dog owner is unwilling to punish her dog because she believes her dog has human feelings. Explanation for Incorrect Answer D : Choice (D) is incorrect. "A zoologist who places the interests of people before those of animals" would not be guilty of the sin mentioned in line 58 ("anthropomorphizing"). Explanation for Incorrect Answer E : Choice (E) is incorrect. A horse trainer's inability to recognize that his horse is hungry has nothing to do with attributing human feelings to animals.

15 Both the author of Passage 1 and the experts mentioned in line 53 of Passage 2 directly support the idea that (A) writers of dog stories intentionally distort the truth for dramatic purposes (B) comparing the intelligence of dogs to that of chimps is a pointless enterprise (C) many people have an excessive emotional attachment to their dogs (D) dogs are less intelligent than many people believe (E) few people are familiar with learning theory as it applies to dogs ANSWERS AND EXPLANATIONS

Explanation for Correct Answer D : Choice (D) is correct. The author of Passage 1 asserts that many people like to believe that dogs have "high-level mental abilities, whether they have earned those extra intelligence points or not." The "experts" mentioned in Passage 2 contribute to the argument that people exaggerate canine intelligence by claiming that dogs "have no real reasoning ability." Explanation for Incorrect Answer A : Choice (A) is incorrect. Passage 2 implies that Meek tried to keep his canine characters "believable," though he may not have always succeeded, and the author of Passage 1 does not mention fiction writers at all. Explanation for Incorrect Answer B : Choice (B) is incorrect. Both the author of Passage 1 and the "experts" cited in Passage 2 seem to find the comparison between "the intelligence of dogs" and "that of chimps" to be worthwhile. Explanation for Incorrect Answer C : Choice (C) is incorrect. Unlike the author of Passage 1, the "experts" do not mention the "emotional attachment" of people to dogs.

Explanation for Incorrect Answer E : Choice (E) is incorrect. Although the "experts" cited in Passage 2 specialize in "learning theory," there is no mention of how familiar people are with their field of study. 16 Based on lines 63-67 (nobody . . . sheep), the author of Passage 2 would most likely appear to the author of Passage 1 as (A) a neutral observer of animal behavior (B) well informed concerning research into animal intelligence (C) having a deep fondness for border collies and therefore overestimating them (D) having little respect for traditional scientific research (E) having a narrow understanding of what constitutes intelligence ANSWERS AND EXPLANATIONS

Explanation for Correct Answer C : Choice (C) is correct. The author of Passage 1 concedes that "dogs can be noble, charming, affectionate, and reliable" but suggests that they may not have "earned those extra intelligence points." In comparison, the description of "pure intelligence shining in the face of a border collie" in lines 6367 may be interpreted to be an overestimation based on the writer's fondness for dogs. Explanation for Incorrect Answer A : Choice (A) is incorrect. The author of Passage 1 would most likely consider the author of Passage 2 to be emotionally attached to dogs and therefore incapable of assessing their intelligence objectively. Explanation for Incorrect Answer B :

Choice (B) is incorrect. Lines 6367 reflect a subjective perspective on canine intelligence. They do not suggest that the author of Passage 2 is particularly knowledgeable about research on the topic. Explanation for Incorrect Answer D : Choice (D) is incorrect. The author of Passage 2 seems to eschew the results of scientific studies in favor of personal experience, but lines 6367 do not reflect a disrespect for "traditional" research. Explanation for Incorrect Answer E : Choice (E) is incorrect. The two writers may have differing views regarding the extent to which dogs are capable of reasoning, but it would be extreme to claim that the author of Passage 2 has a limited comprehension "of what constitutes intelligence." 17 In Passage 2, lines 67-68 (Granted . . . chimp) principally serve to (A) acknowledge a flaw in a prevalent theory (B) digress from a primary claim (C) evoke an air of mystery (D) dismiss a scientific hypothesis as unfounded (E) anticipate a potential objection to an argument ANSWERS AND EXPLANATIONS

Explanation for Correct Answer E : Choice (E) is correct. In admitting that chimps are superior to canines in some traditional measures of intelligence, the author anticipates a possible objection to the argument that dogs possess an unmatched ability to work and communicate with people. Explanation for Incorrect Answer A :

Choice (A) is incorrect. A chimp's ability to navigate a maze faster than a dog can does not contradict any theories mentioned in the passage. Explanation for Incorrect Answer B : Choice (B) is incorrect. A "digression" is a departure from the main argument in a text. Lines 6768, however, are very much related to the author's claim that dogs possess a keen intelligence. Explanation for Incorrect Answer C : Choice (C) is incorrect. On the contrary, the author's use of concession in Passage 2 is meant to clarify the argument made in favor of canine intelligence. Explanation for Incorrect Answer D : Choice (D) is incorrect. Lines 6768 make no attempt to discredit a scientific theory. Instead, they attempt to place the results of a specific scientific finding in context. 18 The authors of both passages mention chimpanzees in order to (A) suggest that the public has a distorted view of chimpanzee intelligence

(B) compare the emotions of primates to those of dogs (C) justify the beliefs of the public regarding the intelligence of certain animals (D) criticize an eccentric scientific claim about animal intelligence (E) provide an example of an animal considered to be highly intelligent ANSWERS AND EXPLANATIONS

Explanation for Correct Answer E :

Choice (E) is correct. The author of Passage 1 suggests that chimpanzees are more intelligent than dogs, while the author of Passage 2 argues that the two animals exhibit different types of advanced intelligence. Explanation for Incorrect Answer A : Choice (A) is incorrect. The author of Passage 2 does not discuss how "the public" views chimpanzee intelligence. Explanation for Incorrect Answer B : Choice (B) is incorrect. Both authors refer to chimpanzees' reasoning abilities, but neither discusses the emotions of primates. Explanation for Incorrect Answer C : Choice (C) is incorrect. The author of Passage 1 asserts that people have a generally flawed understanding of animal intelligence, and the author of Passage 2 doesn't discuss public opinion at all. Explanation for Incorrect Answer D : Choice (D) is incorrect. Neither author mentions chimpanzees in reference to "an eccentric scientific claim." 19 How do the authors of the two passages differ in their assumptions about animal intelligence? (A) The author of Passage 1 implies that intelligence is a single entity, whereas the author of Passage 2 suggests that intelligence can be demonstrated in many distinct ways. (B) The author of Passage 1 believes that no animal can be considered truly intelligent, whereas the author of Passage 2 celebrates the reasoning power of all animals. (C) The author of Passage 1 believes that intelligence can be measured, whereas the author of Passage 2 believes that such quantification would be unethical.

(D) The author of Passage 1 suggests that intelligence is innate, whereas the author of Passage 2 argues that it is acquired. (E) The author of Passage 1 considers intelligence to be developed over time, whereas the author of Passage 2 shows that it is largely static. ANSWERS AND EXPLANATIONS

Explanation for Correct Answer A : Choice (A) is correct. The author of Passage 1 seems to recognize a single form of intelligence by which all animals may be evaluated. The author of Passage 2, however, argues that dogs exhibit an intelligence that is different from that of chimpanzees, but no less useful. Explanation for Incorrect Answer B : Choice (B) is incorrect. Both authors clearly believe that animals can possess intelligence. The author of Passage 1 alludes to the "cleverness of a highly intelligent primate." Explanation for Incorrect Answer C : Choice (C) is incorrect. At no point does the author of Passage 2 delve into the ethics of measuring animal intelligence. Explanation for Incorrect Answer D : Choice (D) is incorrect. Neither author directly discusses assumptions about whether intelligence is "innate" or "acquired." Explanation for Incorrect Answer E : Choice (E) is incorrect. Neither author argues that intelligence is a static or dynamic trait in animals.

Section 10: Writing

1 The sales assistant arranged the gems on the counter, he proceeded to tell us about the origins of each stone. (A) The sales assistant arranged the gems on the counter, he (B) The gems, which were arranged on the counter by the sales assistant, who (C) The gems were first arranged on the counter by the sales assistant, then (D) After arranging the gems on the counter, the sales assistant (E) The sales assistant, having arranged the gems on the counter, he ANSWERS AND EXPLANATIONS

Explanation for Correct Answer D : Choice (D) is correct. It avoids the error of the original by reducing the first complete thought to a phrase. Explanation for Incorrect Answer A : Choice (A) involves improper coordination. Two complete thoughts ("The sales...counter and "he...stone") are joined by only a comma. Explanation for Incorrect Answer B : Choice (B) creates a fragment. There is no main verb in the sentence. Explanation for Incorrect Answer C : Choice (C) creates an illogical sentence. It is illogical that "gems" "proceeded to tell us" something. Explanation for Incorrect Answer E : Choice (E) involves an error in pronoun use. The pronoun "he" is unnecessary.

2 A whistle-blower is when an employee reports fraud or mismanagement in a company. (A) when an employee reports fraud or mismanagement (B) an employee who reports fraud or mismanagement (C) reporting by an employee of fraud or mismanagement (D) if an employee reports fraud or mismanagement (E) fraud or mismanagement being reported by an employee ANSWERS AND EXPLANATIONS

Explanation for Correct Answer B : Choice (B) is correct. It avoids the error of the original by correctly referring to a whistle-blower as "an employee." Explanation for Incorrect Answer A : Choice (A) creates an illogical sentence. A whistle-blower is a what, not a when. Explanation for Incorrect Answer C : Choice (C) creates an illogical sentence. A whistle-blower is a person, not "reporting." Explanation for Incorrect Answer D : Choice (D) creates an illogical sentence. A whistle-blower is a what, not an "if." Explanation for Incorrect Answer E : Choice (E) creates an illogical sentence. It calls a whistle-blower "fraud or mismanagement." 3

After Eliza, the heroine of Shaws Pygmalion, is transformed from a flower girl into a gentlewoman, she realizes that ones social class matters less than your character. (A) she realizes that ones social class matters less than your (B) she realizes that ones social class matters less than ones (C) then realizing that ones social class matters less than their (D) having realized how social class matters less than (E) there is her realization about how social class matters less than ANSWERS AND EXPLANATIONS

Explanation for Correct Answer B : Choice (B) is correct. It avoids the error of the original by using the pronoun "one" consistently. Explanation for Incorrect Answer A : Choice (A) involves a pronoun shift. The pronoun changes from "one" to second-person "your" for no reason. Explanation for Incorrect Answer C : Choice (C) creates a fragment. There is no main verb in the sentence. Explanation for Incorrect Answer D : Choice (D) creates a fragment. There is no main verb in the sentence. Explanation for Incorrect Answer E : Choice (E) involves an error in pronoun reference. "There" does not stand for anything in the sentence. 4 Knowing the roots of words that are hard to spell helps students to become a better speller.

(A) helps students to become a better speller (B) is helpful to students who want to be a better speller (C) helps students to become better spellers (D) is helpful to students in becoming a better speller (E) helps a student be better spellers ANSWERS AND EXPLANATIONS

Explanation for Correct Answer C : Choice (C) is correct. It avoids the error of the original by using plural nouns consistently ("students," "spellers"). Explanation for Incorrect Answer A : Choice (A) involves an error in agreement. The singular noun "a better speller" cannot correctly refer to the plural noun "students." Explanation for Incorrect Answer B : Choice (B) involves an error agreement. The singular noun "a better speller" cannot correctly refer to the plural noun "students." Explanation for Incorrect Answer D : Choice (D) involves an error in agreement. "Students" cannot logically become "a better speller." Explanation for Incorrect Answer E : Choice (E) involves an error in agreement. The plural noun "better spellers" cannot correctly refer to the singular noun "a student." 5 Most experts believe that young childrens not being given physical affection, this interferes with their normal development. (A) young childrens not being given physical affection, this interferes

(B) for young children who have had physical affection withheld from them, it interferes (C) the failure at giving young children physical affection would interfere (D) when withholding physical affection from young children, it interferes (E) the withholding of physical affection from young children interferes ANSWERS AND EXPLANATIONS Explanation for Correct Answer E : Choice (E) is correct. It avoids the error of the original by not separating the subject ("the withholding...children") from its verb ("interferes"). Explanation for Incorrect Answer A : Choice (A) involves incorrect punctuation. The comma unnecessarily separates the subject ("young...affection") from its verb ("interferes"). Explanation for Incorrect Answer B : Choice (B) involves unclear pronoun reference. What the pronoun "it" refers to is not clear. Explanation for Incorrect Answer C : Choice (C) involves improper use of a definite article. The phrase "the failure" suggests that the failure has been referred to previously, but it has not. Explanation for Incorrect Answer D : Choice (D) involves improper phrasing. The verb "withholding" has no subject. 6 Electronic bulletin boards, combining the convenience of a telephone with the massive information storage capacity of a computer, present messages on diverse subjects as astronomy, artificial intelligence, and skydiving. (A) (B) diverse subjects as diverse subjects that are

(C) subjects of such diversity as (D) subjects as diverse as (E) a subject as diverse as ANSWERS AND EXPLANATIONS

Explanation for Correct Answer D : Choice (D) is correct. It avoids the error of the original by using the idiomatic "as . . . as" construction to indicate the variety of subjects addressed on electronic bulletin boards. Explanation for Incorrect Answer A : Choice (A) involves improper phrasing. "As" requires "such" before it. Explanation for Incorrect Answer B : Choice (B) involves improper phrasing. "That are" should be "such as." Explanation for Incorrect Answer C : Choice (C) involves an improper idiom. "Such diversity as" should be "diversity such as." Explanation for Incorrect Answer E : Choice (E) creates an error in agreement (in number). The phrase "a subject" cannot refer to the multiple subjects named. 7 Free from British rule after the American Revolution, a strong central government was an idea that many of the representatives attending the Constitutional Convention were wary of. (A) a strong central government was an idea that many of the representatives attending the Constitutional Convention were wary of

(B) the idea of a strong central government made wary many of the representatives attending the Constitutional Convention (C) many of the representatives attending the Constitutional Convention were wary of a strong central government (D) many representatives at the Constitutional Convention felt wary toward a strong central government (E) many representatives at the Constitutional Convention, wary of a strong central government ANSWERS AND EXPLANATIONS Explanation for Correct Answer C : Choice (C) is correct. It avoids the error of the original by making clear that it was "the representatives" who were "free from British rule." Explanation for Incorrect Answer A : Choice (A) creates an illogical sentence. "A strong central government" is not what had become "free from British rule." Explanation for Incorrect Answer B : Choice (B) creates an illogical sentence. "The idea of a strong central government" is not what had become "free from British rule." Explanation for Incorrect Answer D : Choice (D) involves an improper preposition. One is wary of something, not toward something. Explanation for Incorrect Answer E : Choice (E) creates a fragment. There is no main verb in the sentence. 8 Being cleaner and longer-burning compared with bituminous coal, anthracite was the first coal widely used in the United States for both domestic and industrial purposes. (A) Being cleaner and longer-burning compared with

(B) Both cleaner and more longer-burning compared to (C) Cleaner and longer-burning than (D) By burning longer and more clean than (E) Cleaner as well as longer-burning, unlike ANSWERS AND EXPLANATIONS Explanation for Correct Answer C : Choice (C) is correct. It avoids the error of the original by correctly completing the comparison with the word "than." Explanation for Incorrect Answer A : Choice (A) involves improper comparison. "Compared with" should be "than." Explanation for Incorrect Answer B : Choice (B) involves improper comparison. "Compared to" should be "than." Explanation for Incorrect Answer D : Choice (D) involves improper modification. To modify the verb "burning," "clean" would have to be "cleanly." Explanation for Incorrect Answer E : Choice (E) involves improper comparison. It lacks "than" and adds the unnecessary word "unlike." 9 At graduation, the speaker assured us that our many courses in the liberal arts had prepared us equally well for the challenges of working and further study. (A) had prepared us equally well for the challenges of working and further study (B) had prepared us equally well for the challenges of work and of further study

(C) has supplied the preparation for challenging work along with further study (D) leaves us prepared for the challenges of work and further study both (E) were the preparation for making the challenges of work or further study easier ANSWERS AND EXPLANATIONS Explanation for Correct Answer B : Choice (B) is correct. It avoids the error of the original by making the two items joined by "and" parallel. "Work" and "further study" are both nouns. Explanation for Incorrect Answer A : Choice (A) involves an error in parallelism. The two items joined by "and" should be parallel, so "working" should be the noun "work." Explanation for Incorrect Answer C : Choice (C) involves an error in agreement. The singular verb "has supplied" does not agree with its plural subject, "our many courses in the liberal arts." Explanation for Incorrect Answer D : Choice (D) involves an error in agreement. The singular verb "leaves" does not agree with its plural subject, "our many courses in the liberal arts." Explanation for Incorrect Answer E : Choice (E) involves awkward phrasing. "Were the preparation for making" should simply be "prepared us for." 10 Modern bluegrass songs, telling of love and despair and celebrating mountain beauty, reflect the genres rural origins. (A) Modern bluegrass songs, telling of love and despair and celebrating mountain beauty,

(B) Modern bluegrass songs through their telling of love and despair and celebrating mountain beauty, (C) Because modern bluegrass songs tell of love and despair and also celebrating mountain beauty, they (D) With modern bluegrass songs that tell of love and despair and celebrate mountain beauty, they (E) Telling of love and despair, modern bluegrass songs celebrating mountain beauty, and they also ANSWERS AND EXPLANATIONS Explanation for Correct Answer A : Choice (A) is correct. It uses correct verb forms and punctuation. Explanation for Incorrect Answer B : Choice (B) involves an error in punctuation. The phrase "through...beauty" should be enclosed in commas. Explanation for Incorrect Answer C : Choice (C) creates an illogical sentence. It suggests that "because modern bluegrass songs tell of love," the songs are therefore rural. Explanation for Incorrect Answer D : Choice (D) involves unclear pronoun reference. What the pronoun "they" refers to is not clear. Explanation for Incorrect Answer E : Choice (E) uses an improper verb form. "Celebrating" should be "celebrate." 11 The fruit fly is often used to study genetic mechanisms, because it reproduces rapidly scientists can observe the effects of experiments on several generations. (A) (B) mechanisms, because it reproduces rapidly mechanisms, since it reproduces rapidly,

(C) (D) (E)

mechanisms, since, with its rapid reproduction, mechanisms; because it reproduces rapidly, mechanisms; then rapid reproduction allows

ANSWERS AND EXPLANATIONS Explanation for Correct Answer D : Choice (D) is correct. It avoids the error of the original by correctly joining two complete thoughts ("The fruit fly...mechanisms" and "because...generations") with a semicolon. Explanation for Incorrect Answer A : Choice (A) involves improper coordination. Two complete thoughts ("The fruit fly...mechanisms" and "because...generations") are joined by only a comma. Explanation for Incorrect Answer B : Choice (B) involves improper coordination. Two complete thoughts ("The fruit fly...mechanisms" and "since...generations") are joined by only a comma. Explanation for Incorrect Answer C : Choice (C) involves incorrect punctuation. The subordinating work "since" should not be preceded by a comma. Explanation for Incorrect Answer E : Choice (E) involves improper phrasing. "Allows" should be followed by "to," not by "can." 12 Benin was the first sub-Saharan African country to experience a civilian coup: they were a regime that was dominated by the armed forces and obliged by citizens to implement democratic reforms.

(A) they were a regime that was dominated by the armed forces and obliged by (B) they had been a regime that was dominated by the armed forces, when they were obliged to (C) it had a regime, armed forces dominating, but then were obliged to (D) armed forces dominated them until this regime were obliged by (E) a regime, dominated by the armed forces, was obliged by ANSWERS AND EXPLANATIONS

Explanation for Correct Answer E : Choice (E) is correct. It avoids the error of the original by using the singular verb "was" to agree with the singular noun "regime." Explanation for Incorrect Answer A : Choice (A) involves an error in agreement. The plural pronoun "they" cannot correctly refer to the singular noun "Benin." Explanation for Incorrect Answer B : Choice (B) involves an error in agreement. The plural pronoun "they" cannot correctly refer to the singular noun "Benin." Explanation for Incorrect Answer C : Choice (C) involves an error in agreement. The plural verb "were" cannot agree with the singular subject "Benin." Explanation for Incorrect Answer D : Choice (D) involves an error in agreement. The plural pronoun "them" cannot correctly refer to the singular noun "Benin." 13 This legend about Admiral Nelson, like other naval heroes, are based only partially on fact.

(A) like other naval heroes, are (B) like those of other naval heroes, are (C) like other naval heroes, is (D) like legends about other naval heroes, are (E) like legends about other naval heroes, is ANSWERS AND EXPLANATIONS Explanation for Correct Answer E : Choice (E) is correct. It avoids the error of the original by comparing "this legend" to "legends about other naval heroes." Explanation for Incorrect Answer A : Choice (A) involves an illogical comparison. It compares "this legend" to "other naval heroes." Explanation for Incorrect Answer B : Choice (B) involves an error in agreement. It uses the plural verb "are" for the singular subject "this legend." Explanation for Incorrect Answer C : Choice (C) involves an illogical comparison. It compares "this legend" to "other naval heroes." Explanation for Incorrect Answer D : Choice (D) involves an error in agreement. It uses the plural verb "are" for the singular subject "this legend." 14 Bats and mosquitoes come out at twilight, and the bats would look for mosquitoes and the mosquitoes would look for people. (A) and the bats would look for mosquitoes and the mosquitoes would look (B) and the bats come to look for mosquitoes while the mosquitoes look

(C) the bats look for mosquitoes and the mosquitoes are looking (D) the bats looking for mosquitoes while mosquitoes would look (E) the bats to look for mosquitoes and the mosquitoes to look ANSWERS AND EXPLANATIONS

Explanation for Correct Answer E : Choice (E) is correct. It avoids the error of the original by using consistent verb forms ("to look"). Explanation for Incorrect Answer A : Choice (A) involves an error in verb tense. It shifts from present tense ("come") to past tense ("would look") for no reason. Explanation for Incorrect Answer B : Choice (B) involves an error in verb form. It shifts from "come to look" to "look" for no reason. Explanation for Incorrect Answer C : Choice (C) involves an error in verb form. It shifts from "look" to "are looking" for no reason. Explanation for Incorrect Answer D : Choice (D) involves an error in verb form. The participle "looking" is an incomplete verb.

You might also like